Уравнение второго закона ньютона: Формула второго закона Ньютона в физике

Содержание

Второй закон Ньютона как дифференциальное уравнение движения

Законы Ньютона и законы сохранения. При выводе уравнений движения или покоя среды возможны два подхода. Первый — метод материальной частицы — заключается в составлении на основе второго закона Ньютона дифференциального уравнения движения (покоя) с последующим его интегрированием такой подход применяется главным образом в гидроаэромеханике. Второй — метод контрольных объемов — использует общие законы механики и физики (законы сохранения) для составления суммарных (интегральных) характеристик движения он характерен для гидравлики.  [c.7]
Рассматривая абсолютное.движение КА, запишем, используя второй закон Ньютона, дифференциальное уравнение движения в векторной форме в виде  [c.53]

Дальше излагается кинетика. Вначале, как обычно, читается введение в динамику законы Ньютона, дифференциальные уравнения движения свободной материальной точки.

Баллистическая задача рассматривается как пример решения второй основной задачи динамики свободной материальной точки.  [c.69]

Таким образом, сформировав модель внешней среды и модель неуправляемого ЛА (т. е. методику расчета ускорений и моментов), перейдем к классу, реализующему динамику ЛА. Как уже отмечалось выше, динамика ЛА определяется в результате решения системы обыкновенных дифференциальных уравнений (ОДУ) первого порядка, которую условно принято разделять на две части уравнения динамики центра масс ЛА (в традиционной терминологии — медленное движение), представляющие собой векторную запись второго закона Ньютона, и уравнения углового движения ЛА ( быстрое движение), представляющие собой векторную запись уравнений Эйлера для жесткого тела.  

[c.225]

Движение системы, состоящей из N материальных точек, в инерциальной системе отсчета, в соответствии со вторым законом Ньютона, описывается дифференциальными уравнениями  [c. 121]

Второй закон Ньютона положен в основу составления систем дифференциальных уравнений движения материальной точки. В связи с этим второй закон Ньютона иногда называют основным законом динамики.  [c.318]

Движение свободной материальной точки определяется системой дифференциальных уравнений, вытекающих из второго закона Ньютона, выражаемого в упрощенной форме равенством (Н1.5Ь). Перепишем это равенство так  [c.419]

Дифференциальные уравнения относительного движения точки. Второй закон Ньютона (13.1) для точки М запишем в  [c.300]

В классической гидродинамике уравнение движения вязкой несжимаемой жидкости записывается в форме дифференциального уравнения Навье — Стокса, которое получается на основе второго закона Ньютона.  [c.262]

Обратим внимание на физическое содержание уравнений (3.8) и (3.9). Они выведены из закона количества движения системы, которая для случая сплошной среды образуется непрерывной совокупностью жидких частиц, составляющих объем W.

Поэтому указанные уравнения можно рассматривать как специфические для жидкой среды формы уравнения количества движения. Но при сделанном предположении о постоянстве массы жидкого объема эти же уравнения можно вывести непосредственно из второго закона Ньютона или принципа Даламбера. Поэтому уравнения (3.8) и (3.9) можно также рассматривать как соответственно интегральную и дифференциальную формы второго закона Ньютона для жидкого объема. При этом левая часть уравнения (3.8) представляет собой суммарную инерционную силу, а правая — сумму действующих на массу жидкости внешних сил. В уравнении (3.9) правая часть выражает произведение массы на ускорение (силу инерции) для единичного объема, а левая — сумму действующих на него массовых и поверхностных сил.  
[c.62]


Дифференциальное уравнение движения выражает собой основной закон динамики (второй закон Ньютона) применительно к движущейся сплошной среде. Идею вывода уравнения движения рассмотрим на элементарном примере движения жидкости между двумя параллельными плоскостями (рис. 12.2). Как и в случае уравнения энергии, ограничимся случаем несжимаемой жидкости (капельная жидкость или газ при умеренной скорости движения).  
[c.272]

Уравнение движения. В классической гидродинамике уравне-нме движения вязкой несжимаемой жидкости записывается в форме дифференциального уравнения Навье—Стокса, которое выводится на основе второго закона Ньютона. В проекции на ось Ох 8 0 уравнение имеет вид  [c.155]

Законы движения реактивных летательных аппаратов основаны на разработанной в физике и теоретической механике теории движения твердого тела с переменной массой. Согласно этой теории, которая покоится на классических втором и третьем законах Ньютона, окончательный вид дифференциального уравнения движения таков  [c.415]

Уравнение движения. В уравнении (2-5а) наряду с температурой t имеются еще три переменные Wx, Wy и Wx. Это говорит о том, что в движущейся жидкости температурное поле зависит еще и от распределения скоростей. Последнее описывается дифференциальным уравнением движения, вывод которого основан на втором законе Ньютона сила равна массе, умноженной на ускорение.  [c.38]

Исследуя движение твердого тела в жидкости, Эйлер фактически вводит новую механическую модель — модель Сплошной среды, основанную на его новой аксиоме Сущность этой аксиомы состоит в том, что второй закон Ньютона, впервые записанный Эйлером в виде трех дифференциальных уравнений движения материальной точки  

[c.187]

Скалярные дифференциальные уравнения движения точки переменной массы были установлены в магистерской диссертации И. В. Мещерского Динамит точка переменной массы . Эта работа была опубликована в Петербурге в 1897 г. В истории развития теоретической механики, и особенно ее приложений, в частности, при изучении движения ракет установление исходных уравнений имеет весьма большое принципиальное значение. Второй закон Ньютона вытекает из уравнений Мещерского как частный случай, если предположить, что масса движущейся точки постоянна во все время движения.

[c.110]

Под действием силы упругости пружин возникает ускорение, которое согласно второму закону Ньютона равно = Учитывая выражение (1.2.1), получим дифференциальное уравнение движения  [c.8]

Наиболее примитивный подход к исследованию движения системы, состоящей из п материальных точек, будет, очевидно, сводиться к рассмотрению движений каждой отдельной точки системы. При таком подходе должны быть определены все силы, действующие на каждую точку системы, в том числе и все силы взаимодействия между точками. Определяя теперь ускорения каждой точки в соответствии с законом Ньютона, получим для каждой точки три скалярных дифференциальных уравнения движения второго порядка или Зп дифференциальных уравнений движения для всей системы. Дальнейшее исследование сведется в первую очередь к исключению лишних неизвестных и затем к интегрированию уравнений. Зачастую оказывается, что движение определяется меньшим числом параметров, чем имеется уравнений.

Поэтому возникает проблема — отыскать такие методы решения задач, которые бы приводили к уравнениям, не содержащим лишних параметров и сразу дающим представление о движении механической системы. Первая такая попытка дать общие методы принадлежит швейцарскому математику и механику Якову Бернулли (1654—1705), который, изучая движение маятника, пытался сводить задачу о движении к задаче о равновесии. Дальнейшее развитие принципа принадлежит Даламберу.  
[c.299]


Дифференциальное уравнение движения точки (Л , т ) записывается на основании второго закона Ньютона в виде  [c.167]

Для того чтобы составить дифференциальное уравнение движения материальной точки в неинерциальной системе координат в форме второго закона Ньютона, необходимо к действующим на точку активным силам и реакциям связей присоединить переносную и кориолисову силы инерции.  [c.153]

Согласно подходу Лагранжа частица жидкости движется по траектории, называемой в прикладной механике жидкости струйкой.

Чтобы вывести уравнения такого движения, надо применить второй закон Ньютона к кубу Ка М) и умножить результат на величину, обратную к его объему. В итоге получится дифференциальное уравнение  [c.22]

Работы Мещерского, посвященные теории движения точки переменной массы, имели в виду главным образом астрономические приложения. Мещерский первый в 1897 г. получил основное дифференциальное уравнение движения точки переменной массы и рассмотрел ряд интересных частных задач. Законы изменения массы, которые Мещерский ввел при исследовании задач небесной механики, известны в астрономической литературе как законы Мещерского . При условии постоянства массы из уравнения Мещерского вытекает второй закон Ньютона.  

[c.38]

Зависимость между действующими силами и изменением вектора количества движения можно установить, исходя из второго закона Ньютона. В самом деле, векторное дифференциальное уравнение движения материальной точки имеет вид  [c. 205]

Уравнения (14) называются дифференциальными уравнениями относительного движения точки. Из этих уравнений видно, что, для того чтобы оставить в качестве основного закона динамики второй закон Ньютона, наблюдатель, связанный с подвижной системой координат, должен к числу заданных сил  [c.272]

Введение реакций связей позволяет записать дифференциальные уравнения движения любой точки в форме второго закона Ньютона — уравнений движения свободной (освобожденной от связей) материальной точки  [c.248]

Движение каждого состава описывается обыкновенным дифференциальным уравнением второго порядка, выражающим второй закон Ньютона. В этом уравнении отражаются все силы, действующие на контейнерный состав. В числе этих сил важное место занимает движущая сила, действующая на состав со стороны газа. Ее значение определяется разностью давлений по обе стороны состава. Давления в газе по разные стороны состава находят решением уравнений неустановившегося движения газа между составами, поэтому рассчитать параметры движения последних в трубопроводе невозможно без определения параметров движения газа в областях между ними.[c.89]

К прямой форме дифференциальных уравнений движения можно прийти, непосредственно пользуясь вторым законом Ньютона для выделенных из системы материальных точек выражая силы упругости через перемещения, можно записать  [c.273]

Тогда на основании второго закона механики Ньютона мы можем написать дифференциальные уравнения движения нашей системы п + 1 точек в следующем виде  [c.338]

Чтобы получить дифференциальные уравнения движения системы материальных точек, нужно выразить составляющие ускорения через координаты движущихся точек, применяя второй закон динамики Ньютона, согласно которому составляющая ускорения точки по любой координатной оси равна сумме составляющих по той же оси всех сил, действующих иа эту точку, поделенной на ее массу. Но это правило справедливо только для неподвижной системы координат и поэтому в нашем случае, где система координат движется вместе с точкой Мо, непосредственно неприменимо.[c.355]

Уравнения движения сплошной среды. Дифференциальные уравнения движения жидкости выводятся исходя из применения второго закона Ньютона к произвольному жидкому объему. Этот закон связывает изменение во времени импульса объема жидкости с системой поверхностных и объемных сил, действующего на него. Векторные уравнения движения элемента сплошной среды связывают поля плотности р, вектора ускорения а и тензора напряжений Т во всех внутренних точках. Они установлены О.Коши (1828 г.) и имеют вид  [c.29]

Основной постулат динамики в форме дифференциального уравнения проясняет связь между определением силы и вторым законом Ньютона. Его суть в том, что все механические движения подчиняются уравнению (6.1), где т — скалярный  [c.82]

Второй закон Ньютона как дифференциальное уравнение движения  [c.27]

Для нахождения кинематического закона движения тела. ii (/) необходимо записать уравнение движения, т. е. второй закон Ньютона в дифференциальной форме в проекции на ось Ох. и найти его решение. Из трех сил, действующих на тело, когда оно находится в некоторой точке траектории с координатой х силы тяжести mg.  [c.114]

Наличие в уравнении (14.5) новых переменных величин ы)х, Щ и свидетельствует о том, что в движущейся жидкости температурное поле зависит от распределения скоростей. Эта зависимость выражается дифференциальным уравнением движения жидкости, известным в курсе гидродинамики под названием уравнения Навье — Стокса. Это уравнение выводится на основании второго закона Ньютона, по которому сила равна массе, умноженной на ускорение.  [c.232]

Второй закон Ньютона совместно с правилом сложения сил позволяет составить дифференциальные уравнения движения точки. Исходя из (2.4), найдем  [c.75]


До сих пор в этом курсе изучение движения сводилось к составлению и исследованию дифференциальР ых уравнений, описывающих это движение. Исходным для дифференциальных уравнений любого вида был второй закон Ньютона, устанавливающий связь между ускорением и величиной действующей силы в этот же момент. Поэтому в основе дифференциальных уравнений, которыми мы пользовались до сих пор, всегда лежали локальные  [c.271]

Рассмотрим метод решения подобных задач для одномерного движения жидкости и выведем дифференциальное уравнение. Для этого выделим, как показано на рис. 187, элемент трубки тока dfxds и применим к нему второй закон Ньютона. Проектируя  [c.336]

Мы видели, что дифференциальное уравнение (84) относительного движения материальной точки имеет тот же вид, что и дифференциальное уравнение движения точки относительно неподвижной системы отсчета различие между этими уравнениями состоит лишь в том, что в уравнение относительного движения, кроме заданных сил и реакций связей, входят еще переносная и кориолисова силы инерции. С другой стороны, в главе 21 мы видели, что все общие теоремы динамики точки (теорема о количестве движения, теорема о моменте количества движения, теорема о кинетической энергии) являются следствием основного дифференциального уравнения динамики точки, выражающего второй закон Ньютона. Отсюда следует, что все эти обпще теоремы применимы и к относительному движению точки, но понятно, что, применяя эти теоремы к относительному движению, мы должны принять во внимание переносную и кориолисову силы инерции. В частности, при решении задач, относящихся к относительному движению точки, нередко приходится пользоваться теоремой о кинетической энергии. Нри составлении уравнения, выражающего эту теорему в относительном движении, необходимо принять во внимание работу переносной и кориолисовой сил инерции на относительном перемещении точки. Но так как ускорение Кориолиса Н7д всегда перпендикулярно к относительной скорости v , то следовательно, работа кориолисовой силы инерции в относительном движении равна нулю, и эта сила в уравнение теоремы о кинетической энергии не войдет. Поэтому это уравнение в дифференциальной форме будет иметь следующий вид  [c.456]

Движение глиссирующего судна. Рассмотрим теперь несколько примеров механических систем, движение которых может быть удовлетворительно описано одним дифференциальным уравнением первого порядка. В качестве первого примера рассмотрим прямолинейное движение глиссирующего судна (без учета килевой и бортовой качки). Уравнение его движения согласно второму закону Ньютона может быть записано в виде  [c.260]

Лоренц-инвариантиая форма дифференциального уравнения движения материальной точки. Обратимся сейчас к законам Ньютона и рассмотрим их применимость для релятивистской области. В соответствии с законом сохранения релятивистского импульса для свободной изолированной материальной точки делаем вывод первый закон Ньютона справедлив для релятивистской области свободная изолированная материальная точка движется равномерно прямолинейно в любой инерциальной системе. Второй закон Ньютона приводит к очевидным противоречиям с положением о существовании предельной скорости движения материальных тел и должен быть специально обобщен для квазирелятивистской области движения.  [c.282]

Для вывода дифференциального уравнения движения воспользуемся законом Ньютона, согласно которому произведение массы материальной точки на ее ускорение равно силе, действуюи ,ей в направлении ускорения. В нашем случае масса колеблющегося тела равна W g, где g — ускорение силы тяжести ускорение тела определяется второй производной перемещения по времени и будет обозначаться через X, силами, действующими на колеблющееся тело, являются действующая вниз сила веса W н сила F натяжения пружины (а), которая действует вверх при положении груза, показанном на рис. 1. Таким образом, в рассматриваемом случае дифференциальное уравнение имеет вид  [c.10]

Заметим, что уравнения движения для поступательного (второй закон Ньютона) и вращательного (уравнение моментов) движений имеют одинаковую структуру с той лишь разницей, что. в уравнении моментов вместо линейного стоит угловое ускорение, вместо суммарной силы – суммарный момент сил, а вместо массы тела – его момент инерции относительно оси вращения. (Такое формальное и смысловое соответстзие величин и формул, описывающих поступательное и вращательное движение тела, можно проследить и далее – см. таблицу на с. 70.) Поэтому для тела, вращающегося относительно оси, можно ставить и решать такие же задачи, что и для движения материальной точки или поступательного движения тела. Например, прямая задача в случае вращательного движения, т.е. нахождение кинематического закона вращения (p t), состоит в решении дифференциального уравнения (19.11) при заданных начальных условиях решения задач о свободных колебаниях пружинного и физического маятников в 36).  [c.65]


Уравнение второго закона ньютона в векторном виде. II закон Ньютона в векторной и координатной форме

Чтобы записать законы Ньютона в векторной форме, мы должны поучиться еще кое-чему и определить вектор ускоре­ния. Этот вектор равен производной по времени вектора скоро­сти, причем легко показать, что его составляющие равны вто­рым производным х, у и z no t:

После этого законы Ньютона можно записать таким образом: или ma = F, (11.13)

m(d 2 r/dt 2)=F (11.14)

Фиг. 11.6. Перемещение частиц за малое время t=t 2 -t 1 ,.

Теперь задача о доказательстве инвариантности законов Нью­тона относительно вращений сводится к следующему: нужно доказать, что а (ускорение) есть вектор; это мы уже сделали. Затем нужно доказать, что F (сила) есть вектор; это мы предпола­гаем. Следовательно, если сила есть вектор, то уравнение (11.13) будет выглядеть одинаково во всех системах координат, ибо нам известно, что ускорение тоже вектор. Запись уравнений в виде, не содержащем явно х, у, z, привлекательна тем, что нам нет необходимости выписывать три уравнения каждый раз, ког­да мы хотим написать законы Ньютона или другие законы фи­зики. Мы записываем то, что выглядит как один закон, хотя фактически, конечно, это три закона для каждой оси системы координат, потому что любое векторное уравнение содержит в себе утверждение, что все составляющие равны.

Тот факт, что ускорение – это скорость изменения вектора скорости, помогает найти ускорение в любых, казалось бы, трудных обстоятельствах. Предположим, например, что части­ца, двигаясь по какой-то сложной кривой (фиг. 11.7), имеет в момент t 1 скорость v 1 , а несколько позже, в момент t 2 , скорость v 2 . Чему равно ускорение? Ответ: ускорение равно разности скоростей, деленной на малый промежуток времени; значит, нужно знать разность скоростей. Как же найти эту разность? Чтобы найти разность двух векторов, проведем вектор через концы векторов v 2 и v 1 , иначе говоря, начертим вектор  в ка­честве разности этих двух векторов. Верно? Нет! Мы можем поступать так только тогда, когда начала векторов расположе­ны в одной точке! Вычитать векторы, приложенные к разным точкам, бессмысленно. Остерегайтесь этого! Чтобы вычесть векторы, нужно начертить другую схему. На фиг. 11. 8 векторы v 1 и v 2 перенесены параллельно и равны их двойникам, изоб­раженным на фиг. 11.7.

Фиг. 11 .7. Криволинейная траек­тория.

Фиг. 11.8, Диаграмма для вычисления ускорения.

Теперь можно поговорить об ускорении. Ускорение, конечно, просто равно v/t. Интересно заметить, что разность скоростей можно разделить на две части: можно представить себе, что ускорение состоит из двух составляющих: v ║ – вектора, параллельного касательной к пути, и вектора v ┴ , перпендикулярного к этой касательной. Эти векторы пока­заны на фиг. 11.8. Касательное к пути ускорение равно, есте­ственно, лишь изменению длины вектора, т. е. изменению вели­чины скорости v:

a ║ =dv/dt. (11.15)

Другую, поперечную составляющую ускорения легко вычис­лить, взглянув на фиг. 11.7 и 11.8. За короткое время t изме­нение угла между v 1 и v 2 равно малому углу . Если величина скорости равна v, то

v ┴ =v, а ускорение а равно

а ┴ =v(d/dt).

Теперь нам нужно знать /t. Эту величину можно найти так: если в данный момент кривую можно приблизительно заменить окружностью радиусом R, то, поскольку за время t частица пройдет расстояние s=vt, изменение угла равно

=v(t/R) или /t=v/R.

Таким образом, как мы уже установили ранее,

Чтобы записать законы Ньютона в векторной форме, мы должны поучиться еще кое-чему и определить вектор ускорения. Этот вектор равен производной по времени вектора скорости, причем легко показать, что его составляющие равны вторым производным и по :

, (11. 11)

(11.12)

После этого законы Ньютона можно записать таким образом:

Теперь задача о доказательстве инвариантности законов Ньютона относительно вращении сводится к следующему: нужно доказать, что (ускорение) есть вектор; это мы уже сделали. Затем нужно доказать, что (сила) есть вектор; это мы предполагаем. Следовательно, если сила есть вектор, то уравнение (11.13) будет выглядеть одинаково во всех системах координат, ибо нам известно, что ускорение тоже вектор. Запись уравнений в виде, не содержащем явно , привлекательна тем, что нам нет необходимости выписывать три уравнения каждый раз, когда мы хотим написать законы Ньютона или другие законы физики. Мы записываем то, что выглядит как один закон, хотя фактически, конечно, это три закона для каждой оси системы координат, потому что любое векторное уравнение содержит в себе утверждение, что все составляющие равны.

Тот факт, что ускорение – это скорость изменения вектора скорости, помогает найти ускорение в любых, казалось бы, трудных обстоятельствах. Предположим, например, что частица, двигаясь по какой-то сложной кривой (фиг. 11.7), имеет в момент , скорость , а несколько позже, в момент , скорость . Чему равно ускорение? Ответ: ускорение равно разности скоростей, деленной на малый промежуток времени; значит, нужно знать разность скоростей. Как же найти эту разность? Чтобы найти разность двух векторов, проведем вектор через концы векторов и иначе говоря, начертим вектор в качестве разности этих двух векторов. Верно? Нет! Мы можем поступать так только тогда, когда начала векторов расположены в одной точке! Вычитать векторы, приложенные к разным точкам, бессмысленно. Остерегайтесь этого! Чтобы вычесть векторы, нужно начертить другую схему. На фиг. 11.8 векторы и перенесены параллельно и равны их двойникам, изображенным на фиг. 11.7. Теперь можно поговорить об ускорении. Ускорение, конечно, просто равно . Интересно заметить, что разность скоростей можно разделить на две части: можно представить себе, что ускорение состоит из двух составляющих: – вектора, параллельного касательной к пути, и вектора , перпендикулярного к этой касательной. Эти векторы показаны на фиг. 11.8. Касательное к пути ускорение равно, естественно, лишь изменению длины вектора, т. е. изменению величины скорости .

Фигура 11.7. Криволинейная траектория.

Фигура 11.8. Диаграмма для вычисления ускорения.

Другую, поперечную составляющую ускорения легко вычислить, взглянув на фиг. 11.7 и 11.8. За короткое время изменение угла между и равно малому углу . Если величина скорости равна , то

Таким образом, как мы уже установили ранее,

Чтобы записать законы Ньютона в векторной форме, мы должны поучиться еще кое-чему и определить вектор ускорения. Этот вектор равен производной по времени вектора скорости, причем легко показать, что его составляющие равны вторым производным х, у и z no t:

После этого законы Ньютона можно записать таким образом:

Теперь задача о доказательстве инвариантности законов Ньютона относительно вращений сводится к следующему: нужно доказать, что а (ускорение) есть вектор; это мы уже сделали. Затем нужно доказать, что F (сила) есть вектор; это мы предполагаем. Следовательно, если сила есть вектор, то уравнение (11.13) будет выглядеть одинаково во всех системах координат, ибо нам известно, что ускорение тоже вектор. Запись уравнений в виде, не содержащем явно х, у, z, привлекательна тем, что нам нет необходимости выписывать три уравнения каждый раз, когда мы хотим написать законы Ньютона или другие законы физики. Мы записываем то, что выглядит как один закон, хотя фактически, конечно, это три закона для каждой оси системы координат, потому что любое векторное уравнение содержит в себе утверждение, что все составляющие равны.

Тот факт, что ускорение — это скорость изменения вектора скорости, помогает найти ускорение в любых, казалось бы, трудных обстоятельствах. Предположим, например, что частица, двигаясь по какой-то сложной кривой (фиг. 11.7), имеет в момент t 1 скорость v 1 , а несколько позже, в момент t 2 скорость v 2 . Чему равно ускорение? Ответ: ускорение равно разности скоростей, деленной на малый промежуток времени; значит, нужно знать разность скоростей. Как же найти эту разность? Чтобы найти разность двух векторов, проведем вектор через концы векторов v 2 и v 1 , иначе говоря, начертим вектор Δ в качестве разности этих двух векторов. Верно? Нет! Мы можем поступать так только тогда, когда начала векторов расположены в одной точке! Вычитать векторы, приложенные к разным точкам, бессмысленно. Остерегайтесь этого! Чтобы вычесть векторы, нужно начертить другую схему.

На фиг. 11.8 векторы v 1 и v 2 перенесены параллельно и равны их двойникам, изображенным на фиг. 11.7. Теперь можно поговорить об ускорении. Ускорение, конечно, просто равно Δv/Δt. Интересно заметить, что разность скоростей можно разделить на две части: можно представить себе, что ускорение состоит из двух составляющих; Δv || — вектора., параллельного касательной к пути, и вектора Δv _|_ , перпендикулярного к этой касательной. Эти векторы показаны на фиг. 11.8. Касательное к пути ускорение равно, естественно, лишь изменению длины вектора, т. е. изменению величины скорости v:

Другую, поперечную составляющую ускорения легко вычислить, взглянув на фиг. 11.7 и 11.8. За короткое время Δt изменение угла между v 1 и v 2 равно малому углу ΔΘ. Если величина скорости равна v, то

ускорение а равно

Теперь нам нужно знать ΔΘ/Δt. Эту величину можно найти так: если в данный момент кривую можно приблизительно заменить окружностью радиусом R, то, поскольку за время Δt частица пройдет расстояние s = vΔt, изменение угла равно
ΔΘ = v.Δt/R, или ΔΘ/Δt = v/R.

Таким образом, как мы уже установили ранее,

Чтобы записать законы Ньютона в векторной форме, мы должны поучиться еще кое-чему и определить вектор ускоре­ния. Этот вектор равен производной по времени вектора скоро­сти, причем легко показать, что его составляющие равны вто­рым производным х, у и z no t:

После этого законы Ньютона можно записать таким образом: или ma = F, (11.13)

m(d 2 r/dt 2)=F (11.14)

Фиг. 11.6. Перемещение частиц за малое время t=t 2 -t 1 ,.

Теперь задача о доказательстве инвариантности законов Нью­тона относительно вращений сводится к следующему: нужно доказать, что а (ускорение) есть вектор; это мы уже сделали. Затем нужно доказать, что F (сила) есть вектор; это мы предпола­гаем. Следовательно, если сила есть вектор, то уравнение (11.13) будет выглядеть одинаково во всех системах координат, ибо нам известно, что ускорение тоже вектор. Запись уравнений в виде, не содержащем явно х, у, z, привлекательна тем, что нам нет необходимости выписывать три уравнения каждый раз, ког­да мы хотим написать законы Ньютона или другие законы фи­зики. Мы записываем то, что выглядит как один закон, хотя фактически, конечно, это три закона для каждой оси системы координат, потому что любое векторное уравнение содержит в себе утверждение, что все составляющие равны.

Тот факт, что ускорение – это скорость изменения вектора скорости, помогает найти ускорение в любых, казалось бы, трудных обстоятельствах. Предположим, например, что части­ца, двигаясь по какой-то сложной кривой (фиг. 11.7), имеет в момент t 1 скорость v 1 , а несколько позже, в момент t 2 , скорость v 2 . Чему равно ускорение? Ответ: ускорение равно разности скоростей, деленной на малый промежуток времени; значит, нужно знать разность скоростей. Как же найти эту разность? Чтобы найти разность двух векторов, проведем вектор через концы векторов v 2 и v 1 , иначе говоря, начертим вектор  в ка­честве разности этих двух векторов. Верно? Нет! Мы можем поступать так только тогда, когда начала векторов расположе­ны в одной точке! Вычитать векторы, приложенные к разным точкам, бессмысленно. Остерегайтесь этого! Чтобы вычесть векторы, нужно начертить другую схему. На фиг. 11. 8 векторы v 1 и v 2 перенесены параллельно и равны их двойникам, изоб­раженным на фиг. 11.7.

Фиг. 11 .7. Криволинейная траек­тория.

Фиг. 11.8, Диаграмма для вычисления ускорения.

Теперь можно поговорить об ускорении. Ускорение, конечно, просто равно v/t. Интересно заметить, что разность скоростей можно разделить на две части: можно представить себе, что ускорение состоит из двух составляющих: v ║ – вектора, параллельного касательной к пути, и вектора v ┴ , перпендикулярного к этой касательной. Эти векторы пока­заны на фиг. 11.8. Касательное к пути ускорение равно, есте­ственно, лишь изменению длины вектора, т. е. изменению вели­чины скорости v:

a ║ =dv/dt. (11.15)

Другую, поперечную составляющую ускорения легко вычис­лить, взглянув на фиг. 11.7 и 11.8. За короткое время t изме­нение угла между v 1 и v 2 равно малому углу . Если величина скорости равна v, то

v ┴ =v, а ускорение а равно

а ┴ =v(d/dt).

Теперь нам нужно знать /t. Эту величину можно найти так: если в данный момент кривую можно приблизительно заменить окружностью радиусом R, то, поскольку за время t частица пройдет расстояние s=vt, изменение угла равно

=v(t/R) или /t=v/R.

Таким образом, как мы уже установили ранее,

Динамика поступательного движения

Преамбула про Ньютона : “В Рождественскую ночь 1642 года, в Англии, в семье фермера средней руки была большая сумятица. Родился мальчик такой маленький, что его можно было выкупать в пивной кружке”.

Родился в деревушке Вулсторп, отец которого умер незадолго до его рождения. Его воспитанием занималась бабушка и видела его будущим фермером. Однако мальчик не чувствовал тяги к сельскому хозяйству и окончив школу начал готовиться к поступлению в университет. Поступил и окончил кембриджский университет уже в 1665, а в 1669-1701 годах возглавлял в нём кафедру.

Его первая работа относилась к оптике. При помощи трехгранной стеклянной призмы он разложил белый свет на семь цветов (в спектр), тем самым доказав его сложность (явление дисперсии), он же изобрел первый телескоп – рефрактор, заменив в телескопе линзы на сферические зеркала. Уже в 1671 г ученый усовершенствовал второй рефрактор, что послужило поводом избрания его в члены Королевского общества.

Его работы относятся к механике, оптике, астрономии, математике, именно он открыл закон всемирного тяготения, развил корпускулярную теорию света, разработал дифференциальное и интегральное исчисление, объяснил особенности движения луны.

а) II закон Ньютона в векторной форме:

б) Связь силы и импульса:

(2)

Вектор результирующей силы равен первой производной от вектора импульса м.т. по времени.

Выражение – называют импульсом силы.

Изменение импульса м.т. зависит от продолжительности действия силы, т.е. зависит не только от величины приложенной силы, но и от времени ее действия.

ОПЫТ:

На рис.1 показано действие импульса силы:

а) время действия мало, поэтому обрывается нижняя нить, т.к. массивное тело не успевает прийти в движение;

б) время действия силы велико, поэтому обрывается верхняя нить, тело уже пришло в движение, т.е. на верхнюю нить стала действовать большая сила.

Изучение второго закона Ньютона с помощью машины Атвуда

Цель работы: проверить второй закон Ньютона.

Оборудование: машин Атвуда, перегрузки, электронный секундомер.

1. Краткая теория

Одним из основных законов всей физики является второй закон Ньютона.

Он гласит: ускорение, которое приобретает материальная точка под действием силы, прямо пропорционально величине этой силы, обратно пропорционально

массе материальной точки и направлено в сторону действия силы:

r

ar = F . (2.1)

m

Если на материальную точку действует несколько сил, то ускорение определяется геометрической сумой всех сил, называемой равнодействующей всех сил:

r           1  n     r

a =       ∑ Fi

m i =1

и направлено в сторону равнодействующей силы.

(2.2)

В том случае, когда силы действует не на материальную точку, а на массивное тело конечных размеров, второй закон Ньютона справедлив для центра масс этого тела. То есть ускорение центра масс тела прямо пропорционально  равнодействующей  всех   сил,   действующих  на   тело,   и обратно пропорционально массе всего тела. Однако обычно тела конечных размеров имеют достаточно малые размеры по сравнению с расстоянием, которое они проходят, поэтому их вполне можно считать материальными точками.

Для проверки второго закона Ньютона нужно, прежде всего, исследовать зависимость ускорения тела от действующей на него силы. Для этого нужно

менять  силу,  оставляя  постоянной  массу  тела.  Для  проверки  зависимости ускорения   от        массы            нужно менять            массу  тела     при      неизменной   силе,

действующей на тело.

Прежде чем приступать к рассмотрению экспериментальной установки рассмотрим вспомогательную задачу: через блок перекинута нить, к концам

которой прикреплены грузы, требуется найти ускорение, с которым движутся грузы.

Для решения задач динамики следует придерживаться следующей схемы:

1. Построить схематический рисунок по задаче, на котором изобразить все силы, действующие на каждое из тел, участвующих в движении.

2. Для каждого из тел, участвующих в движении, записать второй закон

Ньютона в векторной форме (2.2). Особо нужно отметить, что в правую часть формулы (2.2) все силы входят со знаком «+» как векторы.

3. Выбрать систему координат. Система координат может быть выбрана отдельно для каждого тела, участвующего в движении. Для оптимального выбора  системы  координат  оси   координат  следует  направлять  по   тому

направлению,            в          котором          направлено    большинство сил.     Это      позволит уменьшить количество ненулевых проекций сил. Если все силы, действующие на данное тело, параллельны одной прямой (линейная система сил), то систему координат можно выбрать только в виде одной оси, параллельной всем силам, действующим на данное тело. Если все силы, действующие на данное тело, параллельны  некоторой  плоскости  (плоская  система  сил),  то  достаточно выбрать только две оси, лежащие в плоскости, параллельной всем силам, действующим на данное тело. При этом опять оси координат должны быть взаимно перпендикулярны и направлены параллельно как можно большему числу сил. И, наконец, если система сил произвольная, то нужно определять все три оси координат.

4. Спроецировать каждое векторное равенство, соответствующее второму закону Ньютона, на все оси координат. Сначала спроецировать векторное равенство формально, т.е. записать это равенство заново с теми же самыми знаками у слагаемых сил, только убрать символ вектора (стрелочку над обозначением данного вектора) и указать символ проекции (т.е. индекс оси, на которую выполняется проецирование, внизу  справа  около  обозначения вектора). После этого определить знаки проекций, т.е. выразить проекции через модули векторов с указанием конкретного знака проекции, если это возможно.

5.  В  результате  получится  несколько  алгебраических  уравнений,  в которых некоторые параметры оказываются неизвестными. Эти уравнения и

нужно решить как систему относительно неизвестных. Однако часто бывает,

что количество неизвестных больше, чем уравнений. В этом случае система оказывается неопределённой. Для её определения нужно добавить некоторые

уравнения. Эти уравнения могут быть получены из уравнений кинематики, связывающих между собой кинематические характеристики. В качестве добавочных  уравнений  могут  быть  уравнения,  полученные  из  некоторых

дополнительных условий. А именно, если тела связаны невесомыми связями (невесомые нити), то усилия на придание связям ускорения равны нулю. Это значит, что сила натяжения связи на её одном конце равна силе натяжения этой

связи на другом конце. Если связь нерастяжима, то перемещение одного её конца вдоль связи за некоторый промежуток времени равно перемещению её второго конца. Отсюда следует, что и скорость и ускорение концов связи вдоль

этой связи будут одинаковыми.

6. Решить систему полученных уравнений относительно требуемых неизвестных.

Применим эту схему к решению поставленной задачи.

1. Схема задачи представлена на рис. 2.1.

r            r T4     T3 r      r T2      T1

r           r

m2 g

m1 g

Рис.2.1.Силы, действующие на систему тел

На тела действует сила тяжести и сила натяжения нитей. На блок сила натяжения  нитей  и  сила  реакции  со  стороны  оси,  вокруг  которой  блок вращается (на рис. не показана).

2. В данном случае в движении участвуют три тела: два тела, подвешенные к нитям, и блок. Поэтому уравнений Ньютона будет три. Уравнение поступательного движения для первого тела:

r           r           r

для второго тела:

m1a1  = m1 g + T1 ,    (2.3)

r           r           r

m2 a2  = m2 g + T2 ,   (2.4)

и уравнение вращательного движения для блока:

r           r           r

где I  – момент инерции блока,

εr  – угловое ускорение,

Iε = M 3 + M 4 ,       (2.5)

r           r

M 3      M 4

– моменты сил натяжения нитей, действующих на блок.

3. Выберем систему координат. Для тел, подвешенных к нитям, достаточно выбрать одну ось. Направим её вертикально вниз и обозначим ox . Для блока нужно три оси, поскольку угловое ускорение направлено перпендикулярно к рисунку. Ось ox  направим вертикально вниз, oy  – вправо, а ось oz  перпендикулярно к рисунку в сторону наблюдателя.

4. Тогда проекции уравнений (2.3)-(2.5) будут иметь вид

m1a1 x  = m1 g x  + T1 x ,      (2.6)

m2 a2 x  = m2 g x  + T2 x ,     (2.7)

Iε z  = M 3 z  + M 4 z .         (2.8)

Поскольку  направление  ускорения  тел  заранее  неизвестно  (оно  зависит  от соотношения масс тел), то конкретизировать их знаки не будем. Ускорение

свободного падения направлено вниз и совпадает по направлению с осью ox ,

значит, проекция его положительна. Силы  r       r

r

направлены вверх, значит, их

проекции       отрицательны.           Момент          силы

T4 относительно       оси      вращения

направлен к наблюдателю, т.е по оси oz , значит, его проекция положительна, а

r

момент  силы T3

направлен  от  наблюдателя,  против  оси oz ,  поэтому  его

проекции отрицательна. Так что уравнения (2.6)-(2.7) перепишутся по-другому:

m1a1 x  = m1 g − T1 ,

(2. 9)

m2 a2 x  = m2 g − T2 ,

(2.10)

Iε z  = −M 3 + M 4 .

(2.11)

В данных уравнениях мы выразили проекции векторов через их модули, о чём говорит тот факт, что обозначения величин не имеют ни символа вектора, ни символа проекции.

Моменты сил по модулю равны произведению модуля сил на плечо:

M 3  = T3 R,M 4  = T3 R ,      (2.12)

где R есть радиус блока.

Так что вместо третьего уравнения можно использовать уравнение:

Iε z  = T4 R − T3 R .           (2.13)

5. Неизвестными в этой системе уравнений являются следующие: ускорения трёх тел, участвующих в движении; величины сил натяжения нити в начале и в конце прямолинейного участка, таких неизвестных четыре. Всего получается 7 неизвестных, а уравнений только три. Так что требуется ещё 4 уравнения. Для отыскания этих уравнений воспользуемся прежде всего тем фактом, что нить невесома. Тогда силы натяжения на разных концах нитей будут равны:

T3  = T1 ,T4  = T2 ,     (2. 14)

Для отыскания ещё двух уравнений воспользуемся нерастяжимостью нити. Это приводит к равенству модулей ускорений тел. Однако, если одно из тел движется по направлению оси ox , другое – против направления этой оси. Так что их ускорения всегда противоположно направлены, поэтому:

a1 x  = −a2 x .           (2.15)

И последнее уравнение получим из условия отсутствия проскальзывания нити по блоку. Это приводит к тому, что тангенциальное ускорение точек обода блока равно по модулю ускорению нити. Что же касается знака ускорений, то из анализа направлений движения тел можно понять, что если первое тело движется вниз с ускорением, т.е. проекция его ускорения на ось ox положительна, то угловое ускорение блока направлено от наблюдателя, т.е против   оси   oz .   Так   что   проекции   ускорений   первого   тела   и   блока противоположны. Поэтому:

ε z R = −a1 x .         (2.16)

Это и есть недостающее уравнение системы. Выпишем теперь все уравнения:

⎧m1a1x  = m1 g − T1;

⎪m2 a2 x  = m2 g − T2 ;

⎪Iε z  = T4 R − T3 R;

⎪T3  = T1 ,T4  = T2 ;

(2. 17)

⎪   z R = −a1x ;

Из 4-го уравнения силы T3

и T4

⎪⎩a1x  = −a2 x .

подставим в третье:

⎧m1a1x  = m1g − T1;

⎪m2a 2 x  = m2g − T2 ;

⎨Iεz  = T2 R − T1R;

(2.18)

⎪ε R = −a   ;

⎪  z    1x

⎪⎩a1x  = −a 2 x .

Выразим теперь угловое ускорение из предпоследнего и подставим в третье:

⎧m1a1x  = m1 g − T1;

⎪m2 a2 x  = m2 g − T2 ;

⎨    − a1x

⎪I

= T2 R − T1R;

(2.19)

⎪⎩a1x  = −a2 x .

Теперь во втором уравнении заменим проекцию ускорения второго тела через

проекцию ускорения первого тела:

⎧m1a1x  = m1 g − T1;

⎪− m2 a1x  = m2 g − T2 ;

⎨    − a1x

⎪I

= T2 R − T1R;

(2.20)

⎪⎩a1x  = −a2 x .

Теперь из первого уравнения вычтем второе:

⎧(m1  + m2 )a1x  = (m1 − m2 ) g + T2 − T ;1

⎪⎩I

− a1x

R

= T2 R − T1R;

(2. 21)

И, наконец, из последнего уравнения выразим разность сил натяжения нитей и подставим в первое:

( m1

+ m2

)a1 x

= ( m1

− m2

)g − I a1 x  .   (2.22)

R2

В результате получили одно уравнение, в котором только одно неизвестное –

ускорение первого тела. Найдём его:

a1 x

=  ( m1 − m2 )g

m  + m  +  I

1          2          R2

.           (2.23)

Проанализируем эту формулу. Из неё видно:

1.  Если  первое  тело  тяжелее  второго,  проекция  ускорения  первого  тела положительна, т.е. ускорение первого тела направлено вниз;

2. Если первое тело легче второго, его ускорение направлено вверх.

3. Если массы тел равны, то их ускорения равны нулю.

Таким  образом,  величина  ускорения  тел  определяется  разностью  масс  тел.

Поэтому величину

Fуск  = ( m1 − m2 )g

часто называют ускоряющей силой при решении подобных задач.

(2.24)

4. Если моментом инерции блока можно пренебречь, то формула ускорения упрощается:

a1x

= (m1 − m2 ) g .        (2.25)

m1 + m2

Из этой формулы видно, что ускорению тел способствует разность масс, а препятствует их сумма.

Этой формулой и пользуются в данной лабораторной работе, чтобы проверить второй закон Ньютона.

2. Экспериментальная установка

Экспериментальная установка представляет собой машину Атвуда, схема которой представлена на рис. 2.2.

К  нерастяжимой, невесомой,  гибкой  нити,  перекинутой  через  лёгкий блок, прикреплены платформы одинаковой массы (4). На эти платформы могут устанавливаться дополнительные перегрузки различных масс. Если масса одного из перегрузков больше, чем масса другого, обе платформы начинают

ускоренно двигаться. В работе эксперимент выполняется всегда таким образом, что масса правого перегрузка больше массы левого. Поэтому правая платформа движется          вниз,  а  левая  –  вверх.  Для  определения  ускорения  платформ

необходимо  измерить  время  прохождения  платформами  некоторого расстояния. С этой целью лабораторная работа снабжена электронным секундомером,   который    запускается   и    останавливается        по    команде

фотодатчиков 5 и 6. Когда правая платформа проходит мимо фотодатчика 6, секундомер запускается, когда платформа проходит мимо фотодатчика 5, секундомер  останавливается.  Таким   образом,   время,   которое   показывает

секундомер, есть время движения правой платформы от верхнего фотодатчика до нижнего. Расстояние между фотодатчиками измеряется с помощью мерной линейки 3, расположенной за установкой.

1

6

4          4

3

2

5

Рис. 2.2. Машина Атвуда

В исходном состоянии левая платформа касается магнита 2 и удерживается его магнитным полем. При выключении магнита обе платформы начинают   движение.   Для   определения   ускорения   платформ   необходимо

измерить:  1)  расстояние от  нижнего  конца  правой  платформы до  верхнего фотодатчика в исходном состоянии платформ, обозначим его s1 ; 2) расстояние

между фотодатчиками,         обозначим      его

платформы между фотодатчиками.

s2 ;       3)         время  движения       t           левой

Обозначим время движения платформы от начальной точки до первого

фотодатчика

t1 , время движения платформы до второго фотодатчика обозначим t2 .

Тогда можно записать следующие кинематические уравнения:

t 2

s  = a  1   , s

1          2          2

t 2

= a  2   .           (2.26)

2

В этих формулах три неизвестных:

t1 , t2 , g . При этом уравнений только

два. Однако можно использовать ещё тот факт, что разность времени полёта шарика до второго и первого фотодатчиков равна показаниям секундомера t :

t2  − t1  = t .  (2.27)

Выразим из первого уравнения системы (2.26) первое время из второго –

второе и подставим в (2.27):

t1 =

2s1 , t  =

a          2

2s2  .    (2.28)

a

2s2   −

a

2s1

a

= t .      (2.29)

В результате получаем одно уравнение с одним неизвестным. Из него легко найти ускорение платформ:

a = 2

( s  −

t 2

s  )2

.           (2.30)

3. Задания

1. Исследовать зависимость ускорения платформ от действующей силы.

Для      этого  согласно  формуле  (2.25)  необходимо  изменять  разность  масс платформ, не меняя их сумму. Исследование состоит из двух экспериментов. В

первом эксперименте на платформы устанавливают перегрузки такие, чтобы масса правой платформы была или на немного (3-5 г) больше массы левой платформы.  Выполняют  эксперимент  по  измерению  ускорения  платформ.

Причём выполняется эксперимент три раза для увеличения точности. Данные заносят в табл.:

mпр

s1

s2

t

a

1

2

3

По результатам последней колонки найти среднее значение ускорения и

погрешность.

Во       втором            эксперименте            оба      перегрузка     устанавливают          на        правую платформу. В этом случае ускоряющая сила становится больше, а общая масса

обеих  платформ       остаётся          неизменной.  Снова проводят        эксперимент  и заполняют таблицу, подобную предыдущей.

Проверяют отношение среднего значения ускорений во втором эксперименте к среднему значению ускорения в первом эксперименте. Оно должно быть равно в пределах погрешности отношению суммы масс перегрузков к разности их масс.

2. Исследовать зависимость ускорения платформ от их масс при одинаковой силе.  Для  этого  снова  выполняют два  эксперимента. В  первом

эксперименте на платформы устанавливают перегрузки такой массы, чтобы масса правой платформы была на немного (3-5 г) больше массы левой платформы.  Выполняют  эксперимент  с  заполнением  табл.  и  отысканием

среднего значения ускорения платформ.

Во втором эксперименте на каждую платформу добавляют по одинаковому перегрузку. Тем самым суммарная масса платформ возрастает, а

их разность остаётся неизменной. Снова выполняют эксперимент, заполняют таблицу и находят среднее значение ускорения.

Проверяют отношение ускорения в первом эксперименте к ускорению

платформ во втором эксперименте. Это отношение должно быть равно отношению массы платформ во втором эксперименте к массе платформ в первом  эксперименте.  При  этом  для  отыскания  отношения  масс  платформ нужно к массам перегрузков добавить массы самих платформ.

Равенство этих двух отношений должно быть верным в пределах погрешности эксперимента.

3. По результатам предыдущих заданий сделать вывод о справедливости второго закона Ньютона.

Вопросы для отчёта

1. Сформулировать законы механики Ньютона.

2. Что называется инерциальной системой отсчёта?

3. Как влияет на результаты эксперимента трение в блоке?

4. Вывести формулу натяжения нитей в эксперименте. Выяснить, от чего зависит натяжение нитей.

Материал взят из Методические указания к выполнению лабораторных работ по механике для студентов направления 010000 «Физико-математические науки» (А.С. Парахин)

Второй закон Ньютона – Справочник химика 21

    Второй закон Ньютона обычно используют для описания движения материальной точки или системы материальных точек. [c.43]

    Свободные колебания. Рассмотрим свободные колебания упругой линейной консервативной системы с одной степенью свободы (см. рис. 3.1, а). В соответствии со вторым законом Ньютона тх = —Ру, где Ру — сила упругости или восстанавливающая сила, действующая на тело со стороны упругой связи (пружины). Полагая, что Ру = О при X =0, для линейной упругой системы с жесткостью с получим в произвольном положении Ру -.сх, и, следовательно, дифференциальное уравнение движения тела примет вид тх + сх = О или [c.47]


    Выразим массу по второму закону Ньютона  [c. 33]

    По второму закону Ньютона производная по времени от этой последней величины равна действующей силе, т. е. [c.118]

    Методы решения задач динамики. При решении задач динамики механизмов, например при исследовании движения машинного агрегата или отдельных элементов машин, обычно применяют уравнения динамики в одной из трех форм второго закона Ньютона, уравнения кинетической энергии, уравнения Лагранжа второго рода. [c.43]

    Покажем это на примере движения тел, описываемого общим законом механики, — вторым законом Ньютона  [c.70]

    Физическая модель движения жидкости. Рассмотрим равновесие движущейся жидкости, непрерывно распределенной в пространстве (сплошная среда). Движение жидкости происходит под действием массовых (объемных) и поверхностных сил. Прн выводе уравнений за основу возьмем второй закон Ньютона, согласно которому сумма векторов всех сил (силы тяжести, силы от гидростатического давления, а для реальных жидкостей — силы трения), действующих на выделенный элемент жидкости, равна произведению его массы на ускорение.[c.276]

    Учет инерции. Масса жидкости в трубопроводе равна р18, где р — плотность, 8 — площадь поперечного сечения трубопровода, I — его длина. Сила, сообщающая жидкости ускорение, равна Р — Рз)8. Согласно второму закону Ньютона [c.169]

    Из второго закона Ньютона 1 Н =1 кг1 м/с = 1 кг м/с . Единица работы (энергии) — джоуль (Дж) — работа, производимая силой 1 Н при перемещении точки ее приложения на расстояние 1 м в направлении действия силы [c.21]

    Умножая ускорение частицы на ее массу, получим силу, действующую на частицу (второй закон Ньютона) этим силам противодействует сопротивление, которое газ оказывает любому от- [c.242]

    Применяя второй закон Ньютона только для вязкого сопротивления частицы и силы тяжести вдоль оси (т. е. в направлении 2), получаем уравнение движения частицы [c.245]

    Аналогично тому, как было найдено выражение критерия Ньютона, можно путем подобного преобразования соответствующих дифференциальных уравненнй получить выражения других критериев подобия. Проследим последовательность такой операции на примере подобного преобразования второго закона Ньютона. [c.72]

    Согласно второму закону Ньютона, ускорение а, приобретаемое телом, прямо пропорционально силе /, действующей на тело, и обратно пропорционально массе тела т, т. е. уравнение связи будет  [c.28]


    Уравнение (5.1-15) имеет форму второго закона Ньютона. Оно показывает, что скорость изменения количества движения системы равна сумме сил, действующих на нее. Таким образом, [У-л] представляет собой чистую силу, действующую на жидкий элемент со стороны окружающей жидкости. [c.103]

    Согласно второму закону Ньютона имеем  [c.29]

    По второму закону Ньютона. [c.70]

    Согласно второму закону Ньютона элементарное изменение количества движения равно элементарному импульсу силы  [c.37]

    Волновое уравнение для твердого тела выводят [1] путем применения второго закона Ньютона к элементарному объему [c. 15]

    Согласно второму закону Ньютона сила Р равна произведению массы т на ускорение а Р = та. Отсюда размерность ньютона (н) = (кг) м/сек ). [c.6]

    Вследствие такой молекулярной бомбардировки стенка В испытывает действие некоторой суммарной силы f. Согласно второму закону Ньютона приращение импульса равно силе  [c.114]

    На основании второго закона Ньютона [c.74]

    Построение математической модели. Если оседанию противодействует сила трения ах, где а>0 — коэффициент трения, то, согласно второму закону Ньютона, имеем [c.75]

    Уравнения движения могут быть записаны в форме второго закона Ньютона  [c.23]

    Важнейшим свойством вещества является наличие массы, согласно второму закону Ньютона, определяемой как мера инерции. Однако это свойство вещества проявляется только под действием сил внешнего воздействия и поэтому является пассивным. К активным свойствам вещества следует отнести гравитацию и заряд, действие которых связано с явлениями тяготения масс, притяжения (отталкивания) зарядов. Эти свойства указывают на наличие связи между двумя формами материи — веществом и полем. Для характеристики химической природы веществ особенно важен заряд. Нейтрон имеет заряд, [c.9]

    Уравнение Навье — Стокса можно вывести из второго закона Ньютона  [c.22]

    Порядок дифференциального уравнения — это наивысший порядок входящих в него производных, определяемый числом раз дифференцирования зависимой переменной. Уравнение, рассмотренное в предыдущем разделе, — первого порядка, потому что объем V дифференцировался лишь один раз 1). Существуют уравнения более высокого порядка. Например, классическое уравнение второго закона Ньютона является дифференциальным уравнением второго порядка  [c.22]

    Исходными данными при расчете и выборе тормозных устройств служат приведенная к выходному звену масса движущихся частей, скорость в начале торможения, допустимые значения тормозного пути т и давления р в камере вытеснения объемного двигателя. Для вывода расчетных формул воспользуемся вторым законом Ньютона в виде [c.103]

    Следуя второму закону Ньютона и пренебрегая трением поршня [c.34]

    Запишем сначала уравнение движения приведенной к выходному звену гидропривода массы т (уравнение нагрузки). По второму закону Ньютона имеем [c.322]

    Размер производных единиц принимается в соответствии с физическими законами, устанавливающими соответствующую связь между физическими величинами. Так, например, единица силы—ньютон (н) устанавливается на основании второго закона Ньютона как сила, сообщающая покоящейся массе в 1 кг ускорение, равное 1 м1сек . Очевидно, 1 и = 10 дин. [c.22]

    При действии пульсационной электромагнитной силы на единицу объема турбулентного моля жидкости величина пульсаци-онной скорости за время дискретного движения моля изменяется согласно второму закону Ньютона  [c.251]

    VI, а вторые шесть уравнений — уравнения второго закона Ньютона, поскольку производная —ди/дд, есть сила, действуюш,ая вдоль координаты [c. 117]

    При воздействии на твердую частицу массой т, находящуюся в при-электродном слое электролита под отслоившейся изоляцией, осциллирующего поля силы, действующие на последнюю в соответствии с принципами вибрационной механики [15], принято делить на быструю ij/fx) и медленную F(x) составляющие. В том случае, когда амплитуда ч/(х) изменяется по пространственной координате х (случай стоячей волны для быстрой” вибрационной силы), уравнение движения описывается с помощью второго закона Ньютона (в виде, принятом в теоретической механике)  [c.83]

    Другим основным уравненис.м классической физики является второй закон Ньютона. Этот закон выражает связь между ускоре-27  [c.419]


Незаконная теория Ученый нашел новые доказательства нарушения ньютоновской механики: Наука и техника: Lenta.ru

В конце июня в архиве препринтов Корнельского университета появилась заметка, автор которой утверждал, что в Солнечной системе существуют явления, доказывающие возможность нарушения второго закона Ньютона. Ниже подробно изложена суть этой пугающей работы и предыстория ее написания. Сразу уточним, что опасаться краха привычного порядка мироустройства не стоит. Незыблемость классической физики все еще поддерживается темной материей.

Странности галактического масштаба

Первые признаки того, что во Вселенной что-то не так, появились в 1930-е годы. Работавший в США астроном Фриц Цвикки наблюдал несколько галактик, входящих в одно скопление. В самих галактиках не было ничего странного, однако из всех расчетов следовало, что они никак не могут находиться рядом. Галактики должны были давным-давно разлететься в разные стороны. Однако что-то удерживало их вместе. Цвикки предположил, что галактики притягиваются друг к другу за счет существования некоей скрытой массы, недоступной для обнаружения имеющимися у ученых приборами.

Коллеги Цвикки скептически отнеслись к его теории. Ученый вообще слыл эксцентричным чудаком, хотя впоследствии многие его идеи подтвердились. О загадочной скрытой массе Вселенной исследователи вспомнили спустя несколько десятилетий после того, как Цвикки придумал это понятие. Астрономические наблюдения показали, что без “лишних” килограммов не поддаются объяснению многие другие явления, в частности, вращение звезд на окраинах спиральных галактик.

Было постулировано, что скрытая масса (ее еще часто называют темной материей) представляет собой субстанцию, не участвующую в электромагнитном взаимодействии, но проявляющуюся в гравитационных эффектах. Ученые разработали множество теорий, объясняющих свойства темной материи. Однако экспериментально обнаружить нечто, составляющее большую часть Вселенной, астрономам не удавалось.

Подрывание основ

В научных и околонаучных кругах ходит несколько анекдотов про Цвикки. Самая известная история повествует о придуманном астрономом необычном способе борьбы с турбулентными потоками. Такие воздушные завихрения сильно мешают астрономическим наблюдениям. В одну из ночей турбулентные потоки были особенно сильны. Цвикки приказал своему аспиранту достать ружье и расстрелять досадное погодное явление. К недовольству Цвикки, метод не сработал.

В то время пока большая часть исследователей занималась поиском скрытой массы, израильский ученый Мордехай Мильгром (Mordehai Milgrom) предложил совершенно иное объяснение странностей космических объектов. Он предположил, что объяснение кроется не в загадочной темной материи, а в знакомой динамике Ньютона. Все привыкли, что три закона движения всегда выполняются на Земле. Но работает ли ньютоновская механика в далеких галактиках?

Мильгром усомнился в универсальности второго закона Ньютона F = ma, где F – сила, m – масса объекта, a – его ускорение. В случае движения звезд вокруг центра галактики, входящая в уравнение сила F – это сила притяжения. Ученый обратил внимание, что нарушающие закон звезды движутся с очень малыми ускорениями. Исследователь предположил, что в системах, где ускорение падает ниже определенного критического значения a0, выражение F = ma перестает работать. Вместо нее движением звезд начинает управлять формула F = ma2/a0.

Модифицированный закон Ньютона хорошо объяснял обнаруженные астрономами отклонения в движении галактик. Тем не менее, подавляющее число ученых не воспринимали идею Мильгрома как вескую научную теорию. Модифицированная ньютоновская динамика (МоНД), как назвал ее автор, не соответствовала релятивистской теории гравитации (общей теории относительности, ОТО) и теории относительности в целом. Спустя два десятилетия после выхода статьи Мильгрома физик Якоб Беркенштейн предложил релятивистскую модификацию МоНД. Отношение к гипотезе Мильгрома стало чуть менее прохладным, но в научный мейнстрим она так и не вошла.

Чтобы отвоевать место под солнцем, сторонникам МоНД не хватало фактических данных. Теория модифицированной ньютоновской динамики постулирует существование ряда эффектов, проявляющихся при ускорениях ниже критического ускорения a0. Оно составляет приблизительно 10-8 сантиметров на секунду в квадрате. Для сравнения, ускорение свободного падения равно 9,81 метра на секунду в квадрате. Заметить изменения движения объектов такого порядка да еще и на огромных расстояниях на современном уровне развития техники невозможно.

Новая надежда

В конце июня в архиве препринтов Корнельского университета появилась новая статья Мордехая Мильгрома. Автор рассматривает еще один эффект МоНД, который проявляется в Солнечной системе.

Совсем недавно группа астрономов сделала доклад, который вновь заставил ученых вспомнить о подзабытой теории МоНД. Исследователи наблюдали карликовые галактики в Млечном Пути. Все они обращались в одной плоскости и в одном направлении. Такое поведение можно было бы объяснить их происхождением – карликовые галактики могли родиться от столкновения более крупных звездных скоплений. Однако в этом случае рядом с ними не должно быть темной материи, а без нее поведение “крошек” не укладывается в рамки ньютоновской механики.

Если второй закон Ньютона верен, то где-то на полпути между Солнцем и центром Млечного Пути должна существовать область, в которой их гравитационные воздействия взаимно уничтожаются. В том случае, если выводы МоНД верны, в “мертвой зоне” сохранится гравитация. Мильгром рассчитал параметры этой силы и определил, как именно она будет воздействовать на находящиеся в Солнечной системе объекты.

Согласно расчетам ученого, плоскости орбит планет и комет, обращающихся вокруг Солнца, должны медленно изменять свою ориентацию по отношению к звезде. Рисунок оставляемого ими воображаемого следа должен напоминать лепестки ромашки.

Хотя новый эффект должен проявляться под самым носом у астрономов, зафиксировать его они пока не в состоянии. Чтобы увидеть предсказанное МоНД отклонение орбит, необходимо собрать точные данные о движении планет в течение длительного времени. Объем информации о наших ближайших соседях можно с натяжкой назвать удовлетворительным, а вот данных о траекториях удаленных планет катастрофически не хватает.

Появление новой работы не обеспечит МоНД признания в ближайшее время. Но если раньше ее сторонники должны были отстаивать свою точку зрения без надежды на экспериментальное подтверждение, то теперь у них появился крошечный шанс. Большинство ученых по-прежнему верит в незыблемость физических основ и наличие темной материи. Если в конце концов физикам удастся объяснить явления окружающего мира с их помощью – необходимость в МоНД отпадет сама по себе. Но до тех пор пока этого не произошло, МоНД имеет право на существование как одна из гипотез.

5.4: Второй закон Ньютона – Physics LibreTexts

Второй закон Ньютона тесно связан с его первым законом. Он математически дает причинно-следственную связь между силой и изменениями в движении. Второй закон Ньютона является количественным и широко используется для расчета того, что происходит в ситуациях, связанных с силой. Прежде чем мы сможем записать второй закон Ньютона в виде простого уравнения, которое дает точное соотношение между силой, массой и ускорением, нам нужно уточнить некоторые идеи, о которых мы упоминали ранее.

Сила и ускорение

Во-первых, что мы подразумеваем под изменением движения? Ответ заключается в том, что изменение движения эквивалентно изменению скорости. Изменение скорости по определению означает наличие ускорения. Первый закон Ньютона гласит, что результирующая внешняя сила вызывает изменение движения; таким образом, мы видим, что результирующая внешняя сила вызывает ненулевое ускорение .

Мы определили внешнюю силу в Силах как силу, действующую на объект или систему, которая возникает вне объекта или системы.Рассмотрим это понятие дальше. Интуитивное представление о внешнем верно — оно находится вне интересующей нас системы. Например, на рисунке \(\PageIndex{1a}\) система, представляющая интерес, представляет собой автомобиль и человека внутри него. Две силы, приложенные двумя студентами, являются внешними силами. Напротив, между элементами системы действует внутренняя сила. Таким образом, сила, с которой человек в машине держится за руль, является внутренней силой между элементами рассматриваемой системы.Только внешние силы влияют на движение системы в соответствии с первым законом Ньютона. (Внутренние силы уравновешивают друг друга, как объясняется в следующем разделе.) Следовательно, мы должны определить границы системы, прежде чем мы сможем определить, какие силы являются внешними. Иногда система очевидна, тогда как в других случаях определение границ системы является более тонким. Понятие системы является фундаментальным для многих областей физики, как и правильное применение законов Ньютона. Эта концепция неоднократно пересматривалась при изучении физики.

Рисунок \(\PageIndex{1}\): разные силы, действующие на одну и ту же массу, вызывают разные ускорения. а) Два ученика толкают заглохшую машину. Показаны все внешние силы, действующие на автомобиль. (b) Силы, действующие на автомобиль, перенесены на координатную плоскость (диаграмма свободного тела) для упрощения анализа. (c) Эвакуатор может оказывать большую внешнюю силу на ту же массу и, следовательно, большее ускорение.

Из этого примера видно, что разные силы, действующие на одну и ту же массу, вызывают разные ускорения. На рисунке \(\PageIndex{1a}\) два студента толкают машину с водителем. Показаны стрелки, представляющие все внешние силы. Систему интереса представляет автомобиль и его водитель. Вес \(\vec{w}\) системы и опора на землю \(\vec{N}\) также показаны для полноты и предполагаются сокращающимися (поскольку не было вертикального движения и дисбаланса силы в вертикальном направлении, чтобы создать изменение в движении). Вектор \(\vec{f}\) представляет трение, действующее на автомобиль, и он действует слева, противодействуя движению автомобиля.(Более подробно мы обсудим трение в следующей главе.) На рисунке \(\PageIndex{1b}\) все внешние силы, действующие на систему, складываются вместе, образуя результирующую силу \(\vec{F}_{net} \). Диаграмма свободного тела показывает все силы, действующие на интересующую систему. Точка представляет собой центр масс системы. Каждый вектор силы простирается от этой точки. Поскольку справа действуют две силы, векторы показаны коллинеарно. Наконец, на рисунке \(\PageIndex{1c}\) большая результирующая внешняя сила создает большее ускорение (\(\vec{a}’ > \vec{a}\)), когда эвакуатор тянет машину.

Кажется разумным, что ускорение будет прямо пропорционально и направлено в том же направлении, что и чистая внешняя сила, действующая на систему. Это предположение было проверено экспериментально и показано на рисунке \(\PageIndex{1}\). Чтобы получить уравнение для второго закона Ньютона, мы сначала запишем отношение ускорения \(\vec{a}\) и чистой внешней силы \(\vec{F}_{net}\) как пропорциональность

\[\vec{a}\; \пропто\; \vec{F}_{net}\]

, где символ \(\alpha\) означает «пропорционально».(Вспомните из «Сил», что чистая внешняя сила представляет собой векторную сумму всех внешних сил и иногда обозначается как \(\sum \vec{F}\).) Эта пропорциональность показывает то, что мы сказали словами: ускорение прямо пропорционально к чистой внешней силе. Как только интересующая система выбрана, определите внешние силы и проигнорируйте внутренние. Пренебрежение многочисленными внутренними силами, действующими между объектами внутри системы, такими как мышечные силы в телах учащихся, не говоря уже о бесчисленных силах между атомами в объектах, является огромным упрощением. Тем не менее, это упрощение помогает нам решать некоторые сложные проблемы.

Также кажется разумным, что ускорение должно быть обратно пропорционально массе системы. Другими словами, чем больше масса (инерция), тем меньше ускорение, создаваемое данной силой. Как показано на рисунке \(\PageIndex{2}\), та же внешняя сила, приложенная к баскетбольному мячу, дает гораздо меньшее ускорение, когда она приложена к внедорожнику. Пропорциональность записывается как

\[а\; \пропто\; \фракция{1}{м},\]

, где m — масса системы, а — величина ускорения.Эксперименты показали, что ускорение точно обратно пропорционально массе, точно так же, как оно прямо пропорционально результирующей внешней силе.

Рисунок \(\PageIndex{2}\): Одна и та же сила, действующая на системы с разной массой, создает разные ускорения. а) Баскетболист толкает мяч, чтобы сделать передачу. (Влиянием силы тяжести на мяч пренебречь.) (b) Тот же игрок прикладывает такую ​​же силу к заглохшему внедорожнику и производит гораздо меньшее ускорение. (c) Диаграммы свободного тела идентичны, что позволяет проводить прямое сравнение двух ситуаций.Ряд шаблонов для диаграмм свободного тела появится по мере того, как вы будете решать больше задач и учиться рисовать их в разделе «Рисование диаграмм свободного тела».

Было обнаружено, что ускорение объекта зависит только от суммарной внешней силы и массы объекта. Объединение двух только что данных пропорциональностей дает второго закона Ньютона .

Второй закон движения Ньютона

Ускорение системы прямо пропорционально суммарной внешней силе, действующей на систему, и имеет то же направление, что и обратно пропорционально ее массе.В форме уравнения второй закон Ньютона равен

.

\[\vec{a} = \frac{\vec{F}_{net}}{m},\]

, где \(\vec{a}\) — ускорение, \(\vec{F}_{net}\) — результирующая сила, а m — масса. Это часто записывается в более привычной форме

.

\[\vec{F}_{net} = \sum \vec{F} = m \vec{a}, \label{5.3}\]

, но первое уравнение дает более полное представление о том, что означает второй закон Ньютона. Когда учитываются только величина силы и ускорения, это уравнение можно записать в более простой скалярной форме:

\[\vec{F}_{net} = ma \ldotp \label{5.4}\]

Закон есть причинно-следственная связь между тремя величинами, которая не просто основана на их определениях. Справедливость второго закона основана на экспериментальной проверке. Диаграмма свободного тела, которую вы научитесь рисовать в разделе «Рисование диаграмм свободного тела», является основой для написания второго закона Ньютона.

Пример 5.2: Какое ускорение может создать человек, толкая газонокосилку?

Предположим, что чистая внешняя сила (толчок минус трение), действующая на газонокосилку, равна 51 Н (около 11 фунтов на дюйм).) параллельно земле (рис. \(\PageIndex{3}\)). Масса косилки 24 кг. Каково его ускорение?

Рисунок \(\PageIndex{3}\): (a) Чистая сила, действующая на газонокосилку, направлена ​​вправо на 51 Н. С какой скоростью газонокосилка ускоряется вправо? (b) Показана диаграмма свободного тела для этой задачи.

Стратегия

Эта задача касается только движения в горизонтальном направлении; нам также дана результирующая сила, обозначенная единственным вектором, но мы можем подавить природу вектора и сконцентрироваться на применении второго закона Ньютона.{2} \ldotp\]

Значение

Направление ускорения совпадает с направлением чистой силы, которая параллельна земле. Это результат векторного соотношения, выраженного во втором законе Ньютона, то есть вектор, представляющий результирующую силу, является скалярным множителем вектора ускорения. В этом примере нет информации об отдельных внешних силах, действующих на систему, но мы можем кое-что сказать об их относительных величинах.Например, сила, действующая на человека, толкающего косилку, должна быть больше, чем трение, противодействующее движению (поскольку мы знаем, что косилка движется вперед), а вертикальные силы должны компенсироваться, поскольку в вертикальном направлении не возникает никакого ускорения (косилка движется). только по горизонтали). Найденное ускорение достаточно мало, чтобы быть приемлемым для человека, толкающего косилку. Такое усилие не продлится слишком долго, потому что человек скоро достигнет максимальной скорости.

Упражнение 5.3

На момент спуска на воду HMS Titanic был самым массивным мобильным объектом из когда-либо построенных, его масса составляла 6,0 x 10 7 кг. Если к кораблю приложить силу 6 МН (6 х 10 6 Н), какое ускорение он испытает?

В предыдущем примере мы имели дело с чистой силой только для простоты. Однако на газонокосилку действует несколько сил. Вес \(\vec{w}\) (подробно обсуждаемый в разделе «Масса и вес») тянет косилку вниз к центру Земли; это создает контактную силу на земле.Земля должна воздействовать на газонокосилку направленной вверх силой, известной как нормальная сила \(\vec{N}\), которую мы определяем в Common Forces. Эти силы уравновешены и поэтому не создают вертикального ускорения. В следующем примере мы показываем обе эти силы. Продолжая решать задачи, используя второй закон Ньютона, не забудьте показать множественные силы.

Пример 5.3: Какая сила больше?

  1. Автомобиль, показанный на рисунке \(\PageIndex{4}\), движется с постоянной скоростью. Какая сила больше, \(\vec{F}_{двигатель}\) или \(\vec{F}_{трение}\)? Объяснять.
  2. Та же машина ускоряется вправо. Какая сила больше, \(\vec{F}_{двигатель}\) или \(\vec{F}_{трение}\)? Объяснять.
Рисунок \(\PageIndex{4}\): показан автомобиль, который (а) движется с постоянной скоростью и (б) ускоряется. Как соотносятся силы, действующие на автомобиль, в каждом случае? (a) Что знание о том, что автомобиль движется с постоянной скоростью, говорит нам о чистой горизонтальной силе, действующей на автомобиль, по сравнению с силой трения? (b) Что знание о том, что автомобиль ускоряется, говорит нам о горизонтальной силе, действующей на автомобиль, по сравнению с силой трения?

Стратегия

Мы должны рассмотреть первый и второй законы Ньютона, чтобы проанализировать ситуацию. Нам нужно решить, какой закон применим; это, в свою очередь, расскажет нам о соотношении между силами.

Раствор

  1. Силы равны. Согласно первому закону Ньютона, если результирующая сила равна нулю, скорость постоянна.
  2. В этом случае \(\vec{F}_{engine}\) должно быть больше, чем \(\vec{F}_{friction}\). Согласно второму закону Ньютона, чтобы вызвать ускорение, необходима результирующая сила.

Значение

Эти вопросы могут показаться тривиальными, но на них обычно отвечают неправильно.Чтобы автомобиль или любой другой объект двигался, его необходимо разогнать из состояния покоя до нужной скорости; для этого требуется, чтобы сила двигателя была больше силы трения. Когда автомобиль движется с постоянной скоростью, результирующая сила должна быть равна нулю; в противном случае автомобиль будет ускоряться (набирать скорость). Чтобы решать задачи, связанные с законами Ньютона, мы должны понимать, следует ли применять первый закон Ньютона (где \(\sum \vec{F}\) = \(\vec{0}\)) или второй закон Ньютона (где \(\sum \vec{F}\) не равен нулю). Это станет очевидным, когда вы увидите больше примеров и попытаетесь решить проблемы самостоятельно.

Пример 5.4: Какая ракетная тяга разгоняет сани?

До пилотируемых космических полетов ракетные сани использовались для испытаний самолетов, ракетного оборудования и физиологических воздействий на людей на высоких скоростях. Они состояли из платформы, установленной на одном или двух рельсах и приводившейся в движение несколькими ракетами.

Рассчитайте величину силы, действующей на каждую ракету, называемую ее тягой T, для четырехракетной силовой установки, показанной на рисунке \(\PageIndex{5}\).Начальное ускорение саней 49 м/с 2 , масса системы 2100 кг, сила трения, противодействующая движению, 650 Н.

Рисунок \(\PageIndex{5}\): На сани действует реактивная тяга, которая ускоряет их вправо. Каждая ракета создает одинаковую тягу T. Система здесь — это салазки, их ракеты и их наездник, поэтому никакие силы между этими объектами не учитываются. Стрелка, представляющая трение (\(\vec{f}\)) нарисована больше масштаба.

Стратегия

Несмотря на то, что силы действуют как вертикально, так и горизонтально, мы предполагаем, что вертикальные силы компенсируются, потому что нет вертикального ускорения.Это оставляет нам только горизонтальные силы и более простую одномерную задачу. Направления обозначаются знаками плюс или минус, где право считается положительным направлением. См. диаграмму свободного тела на рисунке \(\PageIndex{5}\).

Раствор

Поскольку ускорение, масса и сила трения заданы, начнем со второго закона Ньютона и будем искать способы найти тягу двигателей. Мы определили направление силы и ускорения как действующие «вправо», поэтому в расчетах нам нужно учитывать только величины этих величин.Следовательно, мы начинаем с

\[F_{net} = ма\]

, где F net — результирующая сила в горизонтальном направлении. Из рисунка видно, что тяги двигателя добавляются, а трение противодействует тяге. {2}) + 650\; N \ldotp\]

Следовательно, общая тяга равна

\[4Т = 1.{5}\; N \ldotp\]

Значение

Цифры довольно большие, поэтому результат может вас удивить. Подобные эксперименты проводились в начале 1960-х годов, чтобы проверить пределы человеческой выносливости, и установка была разработана для защиты людей при аварийном катапультировании реактивного истребителя. Были получены скорости 1000 км/ч с ускорениями 45g. (Вспомним, что g, ускорение свободного падения, равно 9,80 м/с 2 . Когда мы говорим, что ускорение равно 45 g, это 45 x 9.8 м/с 2 , что примерно равно 440 м/с 2 .) Хотя живые объекты больше не используются, с ракетными салазками была получена наземная скорость 10 000 км/ч.

В этом примере, как и в предыдущем, интересующая система очевидна. В более поздних примерах мы видим, что выбор интересующей системы имеет решающее значение, и этот выбор не всегда очевиден.

Второй закон Ньютона — это больше, чем определение; это отношение между ускорением, силой и массой. Это может помочь нам делать прогнозы. Каждая из этих физических величин может быть определена независимо, поэтому второй закон говорит нам нечто основное и универсальное о природе.

Упражнение 5.4

550-килограммовый спортивный автомобиль сталкивается с 2200-килограммовым грузовиком, и во время столкновения результирующая сила, действующая на каждое транспортное средство, равна силе, действующей на другое. Если величина ускорения грузовика равна 10 м/с 2 , какова величина ускорения спортивного автомобиля?

Компонентная форма второго закона Ньютона

Мы разработали второй закон Ньютона и представили его в виде векторного уравнения в уравнении \ref{5.3}. Это векторное уравнение можно записать в виде трех компонентных уравнений:

\[\sum \vec{F}_{x} = m \vec{a}_{x}, \sum \vec{F}_{y} = m \vec{a}_{y}, \ сумма \vec{F}_{z} = m \vec{a}_{z} \ldotp \label{5.5}\]

Второй закон описывает, как тело механически реагирует на окружающую среду. Влияние среды есть результирующая сила \(\vec{F}_{net}\), отклик тела есть ускорение \(\vec{a}\), а сила отклика обратно пропорциональна масса м.Чем больше масса объекта, тем меньше его реакция (ускорение) на воздействие окружающей среды (заданная результирующая сила). Следовательно, масса тела является мерой его инерции, как мы объяснили в Первом законе Ньютона.

Пример 5.5: сила на футбольном мяче

Футбольный мяч массой 0,400 кг перебрасывается через поле игроком; он испытывает ускорение, определяемое выражением \(\vec{a}\) = 3,00 \(\hat{i}\) + 7,00 \(\hat{j}\) м/с 2 . Найдите: а) результирующую силу, действующую на мяч; б) модуль и направление результирующей силы.{o} \ldotp$$

Значение

Мы должны помнить, что второй закон Ньютона является векторным уравнением. В (а) мы умножаем вектор на скаляр, чтобы определить результирующую силу в векторной форме. В то время как векторная форма дает компактное представление вектора силы, она не говорит нам, насколько он «большой» или куда он движется, в интуитивно понятных терминах. В (b) мы определяем фактический размер (величину) этой силы и направление, в котором она движется.

Пример 5.6: Масса автомобиля

Найдите массу автомобиля, если результирующая сила −600,0 \(\hat{j}\) Н создает ускорение −0,2 \(\hat{j}\) м/с 2 .

Стратегия

Деление вектора не определено, поэтому \(m = \frac{\vec{F}_{net}}{\vec{a}}\) не может быть выполнено. Однако масса m является скаляром, поэтому мы можем использовать скалярную форму второго закона Ньютона, \(m = \frac{F_{net}}{a}\).

Раствор

Мы используем m = \(\frac{F_{net}}{a}\) и подставляем величины двух векторов: F net = 600.{2}} = 3000\; кг \ldotp \номер\]

Значение

Сила и ускорение были даны в формате \(\hat{i}\) и \(\hat{j}\), но ответ, масса m, является скаляром и поэтому не дан в \(\hat{ i}\) и \(\hat{j}\) формы.

Пример 5.7

Несколько сил, действующих на частицу На частицу массой m = 4,0 кг действуют силы четырех величин. F 1 = 10,0 Н, F 2 = 40,0 Н, F 3 = 5,0 Н и F 4 = 2.0 N, с направлениями, как показано на диаграмме свободного тела на рисунке \(\PageIndex{6}\). Чему равно ускорение частицы?

Рисунок \(\PageIndex{6}\): Четыре силы в плоскости xy действуют на частицу весом 4,0 кг.

Стратегия

Поскольку это двумерная задача, мы должны использовать диаграмму свободного тела. Во-первых, \(\vec{F}_{1}\) необходимо разложить на x- и y-компоненты. Затем мы можем применить второй закон в каждом направлении.

Раствор

Мы рисуем диаграмму свободного тела, как показано на рисунке \(\PageIndex{6}\).{2},\]

, который представляет собой вектор величиной 8,4 м/с 2 , направленный под углом 276° к положительной оси x.

Значение

В повседневной жизни можно найти множество примеров, когда три или более сил действуют на один объект, например, тросы, идущие от моста «Золотые Ворота», или футболист, которого трое защитников атакуют. Мы видим, что решение этого примера является просто расширением того, что мы уже сделали.

Упражнение 5.5

На автомобиль действуют силы, как показано ниже. Масса автомобиля 1000,0 кг. Дорога скользкая, поэтому трением можно пренебречь. а) Чему равна результирующая сила, действующая на автомобиль? б) Чему равно ускорение автомобиля?

Второй закон Ньютона и импульс

Ньютон фактически сформулировал свой второй закон в терминах импульса: «Мгновенная скорость изменения импульса тела равна суммарной силе, действующей на тело». («Мгновенная скорость» подразумевает, что задействована производная.) Это может быть задано векторным уравнением

\[\vec{F}_{net} = \frac{d \vec{p}}{dt} \ldotp \label{5.6}\]

Это означает, что второй закон Ньютона отвечает на центральный вопрос движения: что вызывает изменение движения объекта? Импульс был описан Ньютоном как «количество движения», способ сочетания скорости объекта и его массы. Мы посвящаем «Линейный импульс» и «Столкновения» изучению импульса.

Пока достаточно определить импульс \(\vec{p}\) как произведение массы объекта m на его скорость \(\vec{v}\):

\[\vec{p} = m \vec{v} \ldotp \label{5.7}\]

Так как скорость является вектором, то и импульс тоже.

Легко визуализировать импульс. Поезд, движущийся со скоростью 10 м/с, имеет больший импульс, чем тот, который движется со скоростью 2 м/с. В повседневной жизни мы называем одну спортивную команду «имеющей импульс», когда она набирает очки против команды соперника.

Если мы подставим уравнение \ref{5.7} в уравнение \ref{5.6}, мы получим

\[\vec{F}_{net} = \frac{d \vec{p}}{dt} = \frac{d (m \vec{v})}{dt} \ldotp\]

Когда m постоянно, мы имеем

\[\vec{F}_{net} = m \frac{d(\vec{v})}{dt} = m \vec{a} \ldotp\]

Таким образом, мы видим, что импульсная форма второго закона Ньютона сводится к форме, данной ранее в этом разделе.

2-й закон Ньютона

2-й закон Ньютона

Второй закон Ньютона

Предположим, что наша система отсчета является инерциальной. Что происходит в инерциальной системе отсчета, когда на объект действует результирующая сила? На этот вопрос отвечают второй и третий законы Ньютона.

Второй и третий закон Ньютона справедливы во всех инерциальные системы отсчета.

Второй закон Ньютона гласит, что ускорение тела прямо пропорционально чистая сила , действующая на него, и обратно пропорциональна его массе.

Неуравновешенные силы вызывают ускорение.
нет результирующей силы <--> нет ускорения

В алгебраической форме мы запишем второй закон Ньютона как F = m a . Это векторное уравнение. То ускорение a = F /м направлено в сторону силы и пропорциональна величине силы. Масса объекта есть мера его инерции , его сопротивление изменению состояния движения.Если два объекта должны одинаковое ускорение, то на более массивный объект должно действовать большая сила, а на менее массивный объект должна действовать меньшая сила. Масса является скалярной величиной.

Единицы: В единицах СИ масса равна измеряется в кг, ускорение в м/с 2 и сила в ньютонах (Н). 1 Н = 1 кг м/с 2 .
(преобразование: 1 фунт-сила = 4,448 Н)

При одном и том же толчке или притяжении большие массы ускоряются меньше, чем меньшие массы.Вам нужно гораздо меньше усилий, чтобы разогнать трехколесный велосипед, чем для того, чтобы разгонять машину. Из-за его инерции вам нужна сила, чтобы ускорить объект. Если на объект не действует результирующая сила, он не будет ускоряться, его скорость не изменится. Если он изначально находился в покое, он и останется в покое, если он движется с заданной скоростью в определенном направлении, он будет продолжать движутся с одинаковой скоростью в одном направлении.

Ссылка: Грузовик и лестница


Проблема:

Суммарная сила F , приложенная к объекту массой m 1 , производит ускорение 3 м/с 2 . Та же сила, приложенная ко второму объект массой m 2 производит ускорение 1 м/с 2 .
(a)  Чему равно отношение m 1 / m 2 ?
(b)  Если m 1 и m 2 объединены, найдите их ускорение под действием силы F .

Решение:

  • Рассуждение:
    Сила постоянна. F = m a . Продукт м и должны оставаться постоянными.
  • Детали расчета:
    (a)  У нас есть F = m 1 a 1 и Ф = м 2 а 2 . Поэтому m 1 a 1 = м 2 а 2 , м 1 2 = а 2 1 = 1/3.
    (б) Теперь F = (м 1 + м 2 ) a = (4/3)m 2 a , так как m 1 = (1/3)m 2 .
    Следовательно, (4/3)m 2 a = m 2 a 2 , a = (3/4)а 2 = 0,75 м/с 2 .
Проблема:

Автомобиль массой 850 кг движется вправо с постоянной скоростью 1,44 м/с.
а) Чему равна полная сила, действующая на автомобиль?
б) Какова общая сила, действующая на автомобиль, если он движется влево с постоянная скорость 1,44 м/с.

Решение:

  • Рассуждение:
    постоянная скорость <--> без ускорения <--> без результирующей силы
  • Детали расчета:
    а) Автомобиль, движущийся вправо с постоянной скоростью, движется с постоянная скорость.Ускорение равно нулю. Суммарная сила на автомобиле равен нулю.
    (b) Автомобиль, движущийся влево с постоянной скоростью, движется с постоянная скорость. Ускорение равно нулю. Суммарная сила на автомобиле равен нулю.

постоянная скорость   без ускорения   без результирующей силы

Проблема:

График зависимости скорости от времени для m = 0,1 кг частица, движущаяся вдоль оси x, начиная с начала координат, показана ниже.
а) Какая результирующая сила действует на частицу в момент времени t = 6 с?
(b) Какова результирующая сила, действующая на частицу в момент времени t = 12 с?
(c) Какая результирующая сила действует на частицу в момент времени t = 16 с?

Решение:

  • Рассуждение:
    Ускорение – это наклон графика зависимости скорости от времени. То сила F = мА.
  • Детали расчета:
    (a) Между t = 5 с и t = 10 с скорость частицы увеличивается с постоянной скоростью от 2 м/с до 7 м/с.
    Ускорение частицы равно a = (5 м/с)/(5 с) = 1 м/с 2 при в любое время между 5 и 10 с.
    Суммарная сила, действующая на частицу, равна F = ma = (0,1 кг)*(1 м/с 2 ) = 0,1 Н в положительном направлении x.
    (b) В любой момент времени между 10 и 15 с скорость частицы равна постоянна, ее ускорение равно нулю, результирующая сила, действующая на частицу, равна нуль.
    (c) Между t = 15 с и t = 19 с скорость частицы уменьшается с постоянной скоростью от 7 м/с до 5 м/с.
    Ускорение частицы равно a = (-2 м/с)/(4с) = -0,5 м/с 2 в любое время между 15 и 19 с.
    Суммарная сила, действующая на частицу, равна F = ma =( 0,1 кг)*(-0,5 м/с 2) = -0,05 Н. F указывает в отрицательном направлении x.

Второй закон Ньютона

Второй закон Ньютона


Следующая: Третий закон Ньютона Вверх: Сила Предыдущий: Первый закон Ньютона

Если 0 (т.е. на тело действует чистая внешняя сила объект), то
Определение : Инерция есть тенденция объекта сопротивляться любому попытаться изменить свое состояние движения. Масса — это сила, необходимая на единицу создаваемого ускорения, и мера инерции. Масса является скаляром и измеряется в килограммах (кг) в системе СИ. Пример: если мяч для боулинга и мяч для гольфа ударить битой, мяч было бы гораздо труднее сдвинуть с места, так как он имеет большую массу и, следовательно, большая инерция.

Примечание:

  • обратно пропорционально м . Это означает, что для при одинаковой силе меньшая масса будет иметь большее ускорение.
  • Второй закон Ньютона представляет собой векторное уравнение, содержащее три скалярных уравнения (в трех измерениях): F х = мА х , F г = мА г , F z = мА z .
  • Первый закон является частным случаем второго закона.
  • Единицей силы в системе СИ является Ньютон (Н). Определение : 1 Ньютон сила, вызывающая ускорение 1 м/с 2 при действии на массу 1 кг. В системе СГС: 1 дина = 1 г см/ с 2 = 10 – 5 Н. В британской инженерная система: 1 фунт (lb) = 4,448 Н.

Определение : Вес ( ) сила, действующая на объект по гравитационное поле.Из второго закона Ньютона

Примечание:
  • Вес представляет собой вектор, направленный к центру Земли, или перпендикулярно земной поверхности.
  • Вес объекта на Земле и на Луне различен, так как сила гравитационного поля разная ( г земля г луна ).
  • Значение г меняется в зависимости от расстояния от центра земли (более об этом в главе 7).Как следствие:
    • Поскольку Земля не является идеальной сферой, вес объекта незначительно меняется от места к месту на земной поверхности.
    • Вес объекта незначительно меняется в зависимости от высоты над земной поверхностью.
  • Для сравнения, масса является скаляром со значением, не зависящим от местоположения. Обратите внимание, однако, что в приближении, что г является постоянным, масса равна пропорциональна величине веса, и две величины могут быть использованы взаимозаменяемо.Это называется принципом эквивалентности.


Следующая: Третий закон Ньютона Вверх: Сила Предыдущий: Первый закон Ньютона
www-admin@theory. uwinnipeg.ca
9/10/1997

Второй закон движения Ньютона: уравнения и примеры | Что такое Второй закон Ньютона? – Видео и стенограмма урока

Что такое второй закон Ньютона?

Второй закон Ньютона гласит, что ускорение объекта зависит от чистой силы, действующей на объект, и массы объекта.

  • Масса определяется как количество материи в объекте.

Два объекта могут быть одинакового размера, но иметь разную массу из-за количества материи в объекте.

  • Скорость определяется как скорость объекта в заданном направлении. Два примера скорости включают автомобиль, движущийся со скоростью 25 миль в час на север, или самолет, летящий со скоростью 575 миль в час в северо-восточном направлении.Объект, который меняет скорость или направление, имеет изменение скорости.
  • Скорость изменения скорости определяется как ускорение . Объект, который меняет направление или скорость, ускоряется. Изменение скорости может увеличиваться или уменьшаться, чтобы считаться ускорением. Например, автомобиль, который поддерживает скорость 25 миль в час, но меняет направление, считается ускоряющимся. Автомобиль, который держит строго на север, но либо увеличивает, либо уменьшает скорость, считается ускоряющимся.
  • Чистая сила определяется как сумма всех сил, действующих на объект. На этом изображении показан объект, движущийся в прямом направлении. {eq}F_{1} {/eq} и {eq}F_{2} {/eq} — векторы, представляющие силы в двух разных направлениях. Чистая сила (F) — это направление, в котором объект будет двигаться с учетом всех сил, действующих на объект.

Чистая сила (F), действующая на этот объект, представляет собой сумму всех других сил (F1 и F2), действующих на объект.

Законы движения Ньютона

Исаак Ньютон был физиком и математиком, наиболее известным своими тремя законами движения.

Первый закон движения Ньютона

Первый закон движения Ньютона гласит, что объект в состоянии покоя или объект в движении останется в покое или в движении, если на него не воздействует неуравновешенная сила.

Сила является величиной с величиной и направлением. Силы часто изображаются стрелками. На этом изображении показаны две силы, действующие на объект. Величина силы представлена ​​длиной стрелки. Суммарная сила, действующая на этот объект, равна нулю, потому что длина стрелок одинакова в обоих направлениях, что представляет собой уравновешенную силу.Согласно первому закону Ньютона, этот объект будет оставаться в покое до тех пор, пока результирующая сила не перестанет уравновешиваться. Неуравновешенная сила – это сила, приложенная в противоположных направлениях, которые не равны по величине или величине.

Силы представлены стрелками. Силы, действующие на этот объект, равны и считаются уравновешенными силами.

Следующее верно для того, чтобы объект считался покоящимся:

  • Объект имеет нулевую скорость.
  • Силы, действующие на объект, уравновешены.
  • Объект не имеет ускорения.

Первый закон движения также называют законом инерции. Инерция описывает

сопротивление объекта изменению состояния движения. Например, слон и кролик бегут с одинаковой скоростью. Слон будет иметь большую инерцию, чем кролик, потому что сила, необходимая для изменения движения слона, будет больше, чем сила, необходимая для изменения движения кролика.

Второй закон движения Ньютона

Второй закон движения Ньютона гласит, что ускорение объекта зависит от общей силы, действующей на объект, и массы объекта. Этот закон представлен уравнением: {eq}F_{net} {/eq}=ma. Уравнение утверждает, что результирующая сила объекта определяется произведением его массы на ускорение.

Третий закон движения Ньютона

Третий закон движения Ньютона гласит, что на каждое действие есть равное и противоположное противодействие. Силы идут парами. Сила, которую этот пловец прикладывает к воде, сочетается с равной и противоположной силой, приложенной водой. Сила воды в ответ на силу, приложенную руками пловца, толкает пловца вперед.

Третий закон Ньютона объясняет силы, с которыми пловец и вода действуют друг на друга.

Второй закон Ньютона гласит, что результирующая сила объекта есть произведение его массы (m) и ускорения (a).{2} {/экв.}). Масса (m) измеряется в килограммах (кг).

Чтобы найти векторную сумму, вычисляется сумма всех сил.

Пример 1:

Объект, толкаемый влево с силой 15 Н и толкаемый вправо с силой 10 Н, будет иметь результирующую силу 5 Н влево.

Пример 2:

На этом изображении показан объект, сила А и сила В которого тянут в противоположных направлениях. Если бы эти силы имели количественные значения, результирующая сила (С) была бы направлена ​​в направлении действия силы А. Это связано с тем, что величина силы А больше силы В.

Суммарная сила этого объекта рассчитывается путем вычитания сил в противоположных направлениях.

Уравнение {eq}F_{net}=ma {/eq} показывает взаимосвязь между чистой силой, массой и ускорением. По мере увеличения массы результирующая сила увеличивается. По мере увеличения ускорения объекта увеличивается и результирующая сила, действующая на объект.Если либо масса, либо ускорение уменьшаются, результирующая сила также уменьшится.

Что такое второй закон Ньютона? – Урок

(0 оценок)

Быстрый просмотр

Уровень: 6 (5-7)

Необходимое время: 1 час

Урок Зависимость:

Тематические области: Физические науки, физика

Ожидаемые характеристики NGSS:


Поделиться:

Резюме

Студенты знакомятся со вторым законом движения Ньютона: сила = масса x ускорение. После обзора силы, типов сил и первого закона Ньютона представлен второй закон движения Ньютона. Обсуждаются как математическое уравнение, так и физические примеры, в том числе машина Этвуда для иллюстрации принципа. Студенты начинают понимать, что ускорение объекта зависит от его массы и силы неуравновешенной силы, действующей на него. Они также узнают, что второй закон Ньютона обычно используется инженерами при проектировании машин, конструкций и продуктов, от башен и мостов до велосипедов, детских кроваток и автоматов для игры в пинбол.Этот урок является вторым в серии из трех уроков, которые должны преподаваться как единое целое. Эта учебная программа по инженерному делу соответствует научным стандартам нового поколения (NGSS).

Инженерное подключение

Второй закон Ньютона служит основой для большей части математики инженерной механики. При изучении динамики инженеры применяют второй закон Ньютона для предсказания движения объекта, на который действует результирующая сила. Используя уравнение F = ma, инженеры могут моделировать положение, скорость и ускорение объекта или измерять эти значения, чтобы узнать о силах, действующих на объект. В области статики инженеры используют второй закон Ньютона для расчета сил, действующих на неподвижные объекты. Поскольку ускорение неподвижного объекта равно нулю, сумма сил, действующих на объект, должна быть равна нулю. Например, при проектировании конструкций инженеры применяют второй закон Ньютона при расчете сил, действующих на соединения каркасов зданий и мостов.

Цели обучения

После этого урока учащиеся должны уметь:

  • Назовите второй закон Ньютона и объясните, что он означает.
  • Объясните, что сила измеряется в ньютонах, а масса измеряется в килограммах.
  • Различать массу и вес.
  • Используйте второй закон Ньютона, чтобы сравнить величину силы, необходимой для перемещения объектов различной массы.

Образовательные стандарты

Каждый урок или занятие TeachEngineering соотносится с одной или несколькими науками K-12, технологические, инженерные или математические (STEM) образовательные стандарты.

Все более 100 000 стандартов K-12 STEM, включенных в TeachEngineering , собираются, поддерживаются и упаковываются сетью стандартов достижений (ASN) , проект D2L (www.достижениястандарты.org).

В ASN стандарты структурированы иерархически: сначала по источнику; напр. по штатам; внутри источника по типу; напр. , естественные науки или математика; внутри типа по подтипу, затем по сортам, и т.д. .

NGSS: научные стандарты следующего поколения — наука
Ожидаемая производительность NGSS

МС-ПС2-2. Спланируйте исследование, чтобы предоставить доказательства того, что изменение движения объекта зависит от суммы сил, действующих на объект, и массы объекта. (6-8 классы)

Согласны ли вы с таким раскладом? Спасибо за ваш отзыв!

Нажмите, чтобы просмотреть другую учебную программу, соответствующую этому ожидаемому результату
Этот урок посвящен следующим аспектам трехмерного обучения NGSS:
Научная и инженерная практика Основные дисциплинарные идеи Концепции поперечной резки
Планируйте расследование индивидуально и совместно, а также в ходе разработки: определяйте независимые и зависимые переменные и элементы управления, какие инструменты необходимы для сбора данных, как будут записываться измерения и сколько данных необходимо для поддержки заявления.

Соглашение о согласовании: Спасибо за отзыв!

Научные знания основаны на логических и концептуальных связях между фактами и объяснениями.

Соглашение о согласовании: Спасибо за отзыв!

Движение объекта определяется суммой действующих на него сил; если общая сила, действующая на объект, не равна нулю, его движение изменится. Чем больше масса объекта, тем большая сила необходима для достижения такого же изменения движения.Для любого данного объекта большая сила вызывает большее изменение движения.

Соглашение о согласовании: Спасибо за отзыв!

Все положения объектов и направления сил и движений должны быть описаны в произвольно выбранной системе отсчета и произвольно выбранных единицах размера. Для того, чтобы поделиться информацией с другими людьми, эти выборы также должны быть разделены.

Соглашение о согласовании: Спасибо за отзыв!

Объяснение стабильности и изменений в естественных или искусственных системах может быть построено путем изучения изменений с течением времени и сил в различных масштабах.

Соглашение о согласовании: Спасибо за отзыв!

Международная ассоциация преподавателей технологий и инженерии – технология
ГОСТ Предложите выравнивание, не указанное выше

Какое альтернативное выравнивание вы предлагаете для этого контента?

Предыстория урока и концепции для учителей

Подготовка учителей

  • Будьте готовы показать учащимся презентацию о силах и втором законе Ньютона (презентация PowerPoint® из 16 слайдов) для проведения урока.
  • (необязательно) Соберите или приобретите машину Этвуда для демонстрации в классе. Машина Этвуда — это просто веревка, натянутая через приподнятый шкив с одинаковым весом на каждом конце веревки; см. слайд 15 для диаграммы. Это относительно легко построить с помощью шкива, веревки и двух бутылок с водой, содержащих равное количество воды. В качестве альтернативы найдите онлайн-симулятор Atwood Machine.
  • Заранее сделайте копии домашнего задания по первому и второму законам Ньютона (по одной на ученика).
  • В какой-то момент во время презентации, возможно, при обсуждении некоторых примеров (слайды 3-7), пройдитесь по тому, как рисовать (концептуальные) векторы диаграммы свободного тела (стрелки) силы, скорости и ускорения, которые учащимся будет предложено нарисовать. сделать в рамках домашнего задания.

Общие понятия

Второй закон Ньютона можно использовать для описания ускорения объекта на основе общей приложенной силы и массы объекта. Уравнение обычно записывается как F=ma . Проще говоря, чем больше сила приложена к объекту, тем быстрее он будет ускоряться. Точно так же, если одна и та же сила приложена к двум объектам разной массы, объект с большей массой будет ускоряться медленнее.

Второй закон движения Ньютона План презентации (слайды 1-16)

Откройте презентацию о силах и втором законе Ньютона, чтобы все учащиеся могли просмотреть и представить содержание урока, руководствуясь приведенным ниже сценарием и текстом в примечаниях к слайду.Слайды анимированы, поэтому при нажатии открывается следующий текст/изображение/ответ.

Цель: Понять, что ускорение объекта зависит как от массы объекта, так и от силы действующей на него силы.

( слайд 2 ) Кратко повторите, что учащиеся должны знать о силах, как описано в предыдущем уроке раздела «Что такое первый закон Ньютона?». Спросите учащихся: Что такое сила? Каковы две категории сил? Каковы семь конкретных типов сил?

( слайды 3-6 ) На этих слайдах показаны картинки, чтобы побудить учащихся подумать о типах сил, которые были представлены в День 1.Для каждого слайда опишите показанную ситуацию и попросите учащихся определить, какая сила проиллюстрирована и является ли она контактной или бесконтактной силой.

( слайд 7 ) Пока учащиеся смотрят на схему, на которой изображены люди, стоящие в разных местах на планете Земля, введите дополнительный тип контактной силы, называемый нормальной силой. Когда вы стоите на земле, ваш вес — это сила, действующая в направлении вниз, а нормальная сила — это то, что мы называем силой земли, отталкивающей вас.Поскольку сила тяжести и нормальная сила уравновешены, ваше движение не изменится.

Сейчас самое время рассмотреть, как рисовать (концептуальные) векторы диаграммы свободного тела (стрелки) силы, скорости и ускорения.

( слайд 8 ) Ожидайте, что учащиеся признают этот слайд очень похожим на слайд из предыдущего урока. Кратко повторите типы сил, представленных в этом уроке, отметив, что в список добавлена ​​нормальная сила.

( слайд 9 ) Повторите определение силы.Свяжите это с первым законом Ньютона, напомнив учащимся, что приложение силы изменяет скорость объекта. Для дальнейшего расширения спросите учащихся: Какая единица используется для измерения силы? В единицах СИ сила измеряется в ньютонах (Н).

( слайд 10 ) Продолжайте повторять первый закон Ньютона и ускорение. Спросите учащихся: что такое первый закон Ньютона? Какие примеры? Каково определение ускорения? Ускорение – это изменение скорости. Ожидайте, что учащиеся уже знают, что сила может вызвать изменение скорости.Предложите учащимся установить связь между тем, что ускорение вызывается силой (ускорение – это изменение скорости).

( слайд 11 ) Познакомить со вторым законом Ньютона: ускорение тела зависит от силы неуравновешенной силы, действующей на него, и массы тела. Это также обычно записывается как F = м а (сила равна массе, умноженной на ускорение).

( слайд 12 ) Если я приложу одинаковую силу к двум объектам, объект с меньшей массой испытает большее ускорение, а более массивный объект испытает меньшее ускорение. Иллюстрация второго закона Ньютона (слайд 12).copyright

Copyright © 2014 RESOURCE Программа GK-12, Инженерный колледж Калифорнийского университета в Дэвисе

Вспоминая предыдущий пример с повозкой, если мы приложим одну и ту же силу к обеим повозкам. Вагон с меньшей массой испытывает большее ускорение, а вагон с большей массой — меньшее ускорение.

В качестве другого примера подумайте о буксире. Чем отличается величина ускорения буксира, когда он тянет большую баржу, по сравнению с тем, когда он ничего не тянет?

( слайды 13-14 ) Что такое масса? Масса – это количество материи, содержащейся в объекте.Затем введите вес как гравитационную силу. Вес — это мера силы гравитации, притягивающей объект. Вес объекта зависит от силы гравитации. Официальный способ записать это так: сила = масса x ускорение. Это эквивалентно силе, равной массе, умноженной на ускорение свободного падения (гравитацию), поскольку оно более доступно для учащихся, которые могут не проводить прямую связь между ускорением и силой тяжести. Скажите учащимся: ускорение является результатом гравитационного поля. На Земле сила из-за гравитации среднего размера, на Луне она крохотная, а на Юпитере она действительно огромна! Обратите внимание, что на каждой планете сила изменяет , как и ускорение из-за силы тяжести (оно зависит от ее массы и радиуса [в квадрате]).В этой истории масса подобна Златовласке, она не меняет размеров, меняется только окружение (среда).

( слайд 15 ) Представьте демонстрацию с использованием машины Этвуда. Начните с удерживания одинаковых масс в разных положениях и попросите учеников предсказать, что произойдет, когда вы отпустите. Добавьте небольшой вес, чтобы учащиеся могли наблюдать, как две массы ускоряются. Когда массы равны, они не движутся, потому что гравитационные силы равны. Когда массы достаточно различны, чтобы преодолеть трение, можно наблюдать ускорение масс.Подчеркните, что разные массы должны иметь разные гравитационные силы, чтобы они могли двигаться. Это приводит ко второму закону Ньютона как подкреплению концепции.

( слайд 16 ) Повторите основные понятия сегодняшнего урока. Завершите презентацию кратким обзором ключевых понятий, перечисленных на слайде, с пропусками для ответов учащихся. На этом этапе ожидайте, что учащиеся поймут, что ускорение объекта зависит от его массы и силы неуравновешенной силы, действующей на него.Второй закон Ньютона является одним из наиболее часто используемых уравнений в технике. Это простое уравнение позволило инженерам проектировать даже самые сложные машины. Это то, что инженеры учитывают и используют при проектировании всего, от башен и мостов до велосипедов, детских кроваток и автоматов для игры в пинбол. Инженеры используют уравнение второго закона для прогнозирования движения объектов и моделирования движения объектов, воспринимающих силы, чтобы определить результирующую силу. Они также используют уравнение для расчета сил, действующих на неподвижные объекты — мы хотим, чтобы силы, действующие на некоторые неподвижные объекты, такие как здания и мосты, равнялись нулю.

Завершите урок выполнением домашнего задания, как описано в разделе «Оценка».

Второй закон Ньютона для вращения

Какое угловое ускорение создается при приложении крутящего момента к объекту? Второй закон Ньютона гласит, что угловое ускорение пропорционально чистому крутящему моменту и обратно пропорционально моменту инерции.

Второй закон Ньютона: Σ τ = I α

Регулируемый момент инерции

Две тяжелые цилиндрические массы размещены на противоположных концах платформы, которая вращается вокруг своего центра.Крутящий момент передается на ось платформы с помощью струны. Струна проходит через шкив, а на другом конце струны свисает груз.

Система выходит из состояния покоя, и платформа начинает вращаться с определенным угловым ускорением.

Если провести второй опыт с перемещением масс ближе к центру платформы, что произойдет?

  1. Угловое ускорение уменьшится.
  2. Угловое ускорение останется прежним.
  3. Угловое ускорение увеличится.

Согласно второму закону Ньютона Σ τ = I α . Крутящий момент от подвешенной массы примерно одинаков в обоих случаях. Перемещение масс ближе к центру уменьшает момент инерции, что увеличивает угловое ускорение.

Применение второго закона Ньютона

Применим второй закон Ньютона к нашей системе двух цилиндрических масс на вращающейся платформе. Определить угловое ускорение системы через:
М, масса каждого цилиндра;
ч — расстояние от оси вращения до каждого цилиндра;
Тл — натяжение струны;
р, радиус оси.

Будем считать, что сама платформа имеет пренебрежимо малую массу, а радиус каждого цилиндра мал по сравнению с h.

Как обычно, начните с диаграммы свободного тела, а затем примените второй закон Ньютона для вращения.

Σ τ = I α

Единственный крутящий момент, о котором мы заботимся, связан с натяжением струны. При крутящем моменте τ = r × F , в этом случае крутящий момент:

τ = rT

Между r и T угол 90 градусов.

Решение второго закона Ньютона для α :

α = rT/I

Для цилиндра, вращающегося вокруг своего центра масс, где ось вращения совпадает с осью цилиндра, момент инерции равен:

I com = ½ MR 2

Каждый из наших цилиндров вращается вокруг параллельной оси (проходящей через центр платформы), а не вокруг своего центра масс. Теорема о параллельной оси говорит нам, что каждый цилиндр имеет инерцию вращения, определяемую выражением:

I 1 = I ком + Mh 2

I 1 = ½ MR 2 + Mh 2

Предположим, что R I 1 = Mh 2

Суммарный момент инерции в два раза больше, потому что цилиндров два:

I = 2Mh 2

Наше выражение для углового ускорения принимает вид:

α = rT/[ 2Mh 2 ]

Числовой пример

Для нашего демонстрационного аппарата у нас есть следующие параметры:
масса каждого цилиндра М = 2. 5 кг
расстояние от оси до каждого цилиндра h = 7,5 см
натяжение струны примерно F = 5 Н
радиус оси r = 2,7 см

α = 0,027 * 5 / (2 * 2,5 * 0,075 2 )

α = 4,8 рад/с 2

Это предсказывает, что первый полный оборот системы занимает 1,6 с.

Повторное посещение машины Этвуда

Машина Этвуда представляет собой устройство, в котором две массы M и m соединены нитью, проходящей через шкив.Предположим, что M > m. Шкив представляет собой сплошной диск массой m p и радиусом r.

Каково ускорение двух масс?

Начните с трех диаграмм свободного тела, по одной для каждой массы и одной для шкива.

Когда мы решали эту задачу раньше, мы предполагали, что шкив не имеет трения и массы, поэтому натяжение везде одинаково. Теперь мы учтем массу шкива, поэтому натяжение будет другим.

Подумайте, что будет делать система.Если систему вывести из состояния покоя, то тяжелая масса будет ускоряться вниз, более легкая – вверх, а шкив будет иметь угловое ускорение по часовой стрелке.

Совместите системы координат с ускорением. У каждого объекта своя система координат, но они должны быть согласованы.
Отнять +y для массы M.
Поднимите +y вверх для массы m.
Поверните по часовой стрелке, чтобы шкив оказался положительным.

Признать, что массы имеют одинаковое ускорение, a.

Примените второй закон Ньютона для каждой массы.

Для массы M: | Для массы m: | Для шкива:
Σ F y = M a y | Σ F у = м а у | Σ τ = I α
Mg – T 1 = Ма | T 2 – мг = ма | RT 1 – RT 2 = ½ м p r 2 α

Две массы дают два уравнения с тремя неизвестными. Шкив дает нам третье уравнение, которое нам нужно. Предполагая, что струна не проскальзывает при контакте со шкивом, ускорение струны а связано с угловым ускорением шкива соотношением:

α = а/р

Тогда уравнение шкива принимает вид:

T 1 – T 2 = ½ м р а

Объединение трех уравнений для устранения двух напряжений дает:

(Mg – Ma) – (mg + ma) = ½ m p a

Mg – мг = Ma + ma + ½ m p a

а = г (М – т) / (М + т + ½ т р )

Это меньшее ускорение, чем a = g (M – m) / (M + m), которое мы получили, пренебрегая шкивом, что имеет смысл.

Числовой пример: M = 210 г и m = 200 г.

а = г (10)/(410) = 0,24 м/с 2 если м р = 0.

Если m p = 200 г, то a = g (10)/(510) = 0,19 м/с 2

Объяснение урока: Второй закон Ньютона: переменная масса

В этом объяснении мы узнаем, как использовать дифференциацию для второго закона Ньютона движения частицы с переменной массой.

Давайте сначала вспомним второй закон движения Ньютона, определив второй закон движения Ньютона для тела постоянной массы.

Определение: Второй закон Ньютона для постоянной массы

Когда на тело действует результирующая сила, тело ускоряется в направлении силы. Величина ускорения зависит от величины силы и от массы тела по формуле 𝐹=𝑚𝑎, где 𝑚 — постоянная масса тела, а 𝑎 есть ускорение тела.

Второй закон движения Ньютона можно также выразить через скорость изменения импульса тела.Импульс тела 𝑝 определяется следующим образом.

Определение: Импульс тела

Импульс тела определяется выражением 𝑝=𝑚𝑣, где 𝑚 — масса тела, а 𝑣 — скорость тела.

Выражение второго закона Ньютона через скорость изменения импульса дает 𝐹=𝑝𝑡=𝑡(𝑚𝑣).dddd

Термины массы и скорости заключены в квадратные скобки, так как любой член может изменяться во времени или быть постоянным, поэтому, возможно, необходимо учитывать скорость изменения любого члена.

Если масса тела, движущегося с ускорением, постоянна, скорость изменения импульса тела определяется выражением 𝐹=𝑡(𝑚𝑣)=𝑚𝑣𝑡=𝑚𝑎, дддд которая представляет собой форму постоянной массы второго закона движения Ньютона. Эквивалентно, для тела, которое движется равномерно, когда на него действует сила, скорость изменения импульса тела определяется выражением 𝐹=𝑡(𝑚𝑣)=𝑣𝑚𝑡.dddd

Предположим, что скорость и масса тела меняются со временем. Если тело увеличивает и скорость, и массу, тогда сила, действующая на тело, должна объяснять как увеличение скорости, так и увеличение массы.

Когда две переменные изменяются по отношению к третьей переменной, можно использовать правило произведения, чтобы дифференцировать произведение двух переменных.

Практическое руководство. Использование правила продукта для дифференциации продукта

Рассмотрим переменные 𝑎 и 𝑏 что оба изменяются во времени. Скорость, с которой 𝑎𝑏 изменяется во времени, определяется выражением ддддд𝑡(𝑎𝑏)=𝑎𝑏𝑡+𝑏𝑎𝑡.

Если мы применим правило произведения к скорости изменения количества движения тела, мы обнаружим, что 𝐹=𝑡(𝑚𝑣)=𝑚𝑣𝑡+𝑣𝑚𝑡.dddddd

Силу, действующую на тело, можно определить следующим образом.

Определение: Второй закон Ньютона в терминах изменения импульса

Когда на тело действует результирующая сила, импульс тела изменяется. Скорость изменения массы и скорости тела зависит от от величины силы и от массы и скорости тела по формуле 𝐹=𝑚𝑣𝑡+𝑣𝑚𝑡, дддд где 𝑚 – масса тела и 𝑣 – скорость тела.

Рассмотрим пример, в котором тело изменяет массу под действием силы.

Пример 1. Нахождение силы, действующей на тело с переменной массой, движущееся с постоянной скоростью

Заполните пропуск: Сила, действующая на массу, изменяющуюся в соответствии с функцией движется с постоянной скоростью 4 м/с.

Ответ

Сила, действующая на тело, вызывает изменение импульса тела: 𝐹=𝑡(𝑚𝑣)=𝑚𝑣𝑡+𝑣𝑚𝑡.dddddd

Тело имеет постоянную скорость, поэтому дд𝑣𝑡=0.

Следовательно, 𝐹=𝑣𝑚𝑡.dd

Утверждается, что значение 𝑣 равно 4 м/с, который можно заменить, чтобы дать 𝐹=4𝑚𝑡.dd

Для силы, действующей на тело для перемещения тела с постоянной скоростью и, следовательно, с нулевой ускорение, масса тела должна изменяться за время действия на него силы.

Вопрос выражает массу тела как функцию времени. Масса в килограммы как функция времени, 𝑚(𝑡), определяется выражением 𝑚(𝑡)=(5+2𝑡), значение которого действительно увеличивается с увеличением 𝑡.

Дифференцирование 𝑚(𝑡) по времени дает дддд𝑚𝑡=𝑡(5+2𝑡)=2.

Масса тела увеличивается на 2 килограмма в секунду.

Скорость изменения массы можно подставить в формулу следующим образом: 𝐹=4𝑚𝑡𝐹=4(2)=8.ddN

Следовательно, тело, движущееся с постоянной скоростью 4 м/с, который начинается для увеличения массы на 2 килограмма в секунду необходимо действовать с силой 8 Н, чтобы поддерживать эту постоянную скорость.

Масса тела не влияет на силу, необходимую для поддержания его равномерное движение, только скорость изменения его массы.

Рассмотрим пример силы, действующей на тело. где и масса, и скорость тела меняются со временем.

Пример 2. Нахождение силы, действующей на тело с переменной массой в любой момент времени

Тело движется прямолинейно. В момент времени 𝑡 секунд, его смещение от фиксированной точки равно 𝑠=6𝑡+9𝑡м. Его масса меняется со временем так, что 𝑚=(8𝑡+9) кг. Запишите выражение для силы, действующей на тело в момент времени 𝑡.

Ответ

Как скорость, так и масса тела меняются со временем, поэтому сила, действующая на тело, находится с помощью 𝐹=𝑚𝑣𝑡+𝑣𝑚𝑡.dddd

Скорость изменения массы тела можно найти дифференцированием зависимость массы тела от времени выглядит следующим образом: дддд𝑚𝑡=𝑡(8𝑡+9)=8.

Масса тела увеличивается на 8 кг в секунду.

Функция скорости изменения скорости тела относительно время не дается вопросом, но дается смещение как функция времени. Скорость изменения смещения равна мгновенной скорости, 𝑣, тела и так далее 𝑣=𝑠𝑡=𝑡6𝑡+9𝑡=(12𝑡+9). dddd

Поскольку 𝑣 известно, мы можем выразить скорость изменения 𝑣 следующим образом: дддд𝑣𝑡=𝑡(12𝑡+9)=12.

Мы можем использовать следующую формулу: 𝐹=𝑡(𝑚𝑣)=𝑚𝑣𝑡+𝑣𝑚𝑡𝐹=((8𝑡+9)(12))+((12𝑡+9)(8))𝐹=(96𝑡+108)+(96𝑡+72)𝐹=(192𝑡+ 180).ddddddN

Сила, изменяющаяся во времени, должна быть выражена как функция 𝑡 и иметь мгновенное значение во все моменты действия силы. Рассмотрим пример, в котором мгновенное значение определяется изменяющаяся во времени сила.

Пример 3. Нахождение силы, действующей на тело с переменной массой в данный момент времени

Тело движется прямолинейно. В момент времени 𝑡 секунд, его смещение от фиксированной точки равно 𝑠=2𝑡+5𝑡+4м. Его масса меняется со временем так, что 𝑚=(6𝑡+5) кг.Определить силу, действующую на тело, когда 𝑡=3с.

Ответ

Чтобы определить силу, действующую в данный момент, должна быть определена функция, представляющая изменение силы во времени.

Как скорость, так и масса тела меняются со временем, поэтому сила, действующая на тело, находится с помощью 𝐹=𝑚𝑣𝑡+𝑣𝑚𝑡. dddd

Скорость изменения массы тела можно найти дифференцированием зависимость массы тела от времени выглядит следующим образом: дддд𝑚𝑡=𝑡(6𝑡+5)=6.

Функция, представляющая скорость тела, определяется выражением 𝑣=𝑠𝑡=𝑡2𝑡+5𝑡+4=(4𝑡+5).dddd

Дифференцирование функции, представляющей 𝑣 по времени, дает нам дддд𝑣𝑡=𝑡(4𝑡+5)=4.

Как произведение 𝑚 и 𝑣, 𝑚𝑣, следует дифференцировать по времени, мы можем использовать правило произведения, чтобы дифференцировать 𝑚𝑣 относительно 𝑡 следующим образом: 𝐹=𝑚𝑣𝑡+𝑣𝑚𝑡𝐹=((6𝑡+5)(4))+((4𝑡+5)(6))𝐹=(24𝑡+20)+(24𝑡+30)𝐹=(48𝑡+50).ddddN

Замена 𝑡=3 дает нам 𝐹=48(3)+50=194.N

Рассмотрим второй закон Ньютона о движении с переменной массой применительно к системе, где скорость задана векторами.

Определение: второй закон Ньютона для переменной массы в терминах векторов ⃑𝐹=𝑚⃑𝑣𝑡+⃑𝑣𝑚𝑡, дддд где и ⃑𝐹, и ⃑𝑣 являются векторными величинами.

Если нам задан вектор смещения вместо скорости, мы можем дифференцировать смещение по времени, чтобы найти скорость, поскольку связь между смещением и скоростью одинакова в векторной и скалярной формах.У нас есть это ⃑𝑣=⃑𝑠𝑡.dd

Теперь мы можем рассмотреть пример использования этой формулы.

Пример 4. Нахождение силы, действующей на тело, с использованием второго закона Ньютона для переменной массы с векторами 5 г/с. Вектор скорости объекта определяется как ⃑𝑣=2𝑡⃑𝑖+12𝑡⃑𝑗/смс. Горизонтальная составляющая силы, действующей на объект, заставляющей его двигаться с этой скоростью, равна динам.

Ответ

Мы знаем, что масса объекта начинается с 10 г и увеличивается со скоростью 5 г/с. Следовательно, мы можем сказать, что 𝑚=10+5𝑡.

Кроме того, поскольку скорость увеличения массы составляет 5 г/с, мы имеем, что дд𝑚𝑡=5.

Чтобы найти силу, действующую на тело, нам нужно будет использовать второй закон Ньютона для переменной массы в терминах векторов. Это говорит нам, что ⃑𝐹=𝑚⃑𝑣𝑡+⃑𝑣𝑚𝑡.dddd

Чтобы найти dd⃑𝑣𝑡, мы просто продифференцируем ⃑𝑣 по 𝑡.Это даст нам дд⃑𝑣𝑡=4𝑡⃑𝑖+24𝑡⃑𝑗.

Теперь у нас есть все компоненты, необходимые для поиска ⃑𝐹. Подставляя в нашу формулу, находим ⃑𝐹 = (10 + 5𝑡) 4𝑡⃑𝑖 + 24𝑡⃑𝑗 + 2𝑡⃑𝑖 + 12𝑡⃑𝑗 × 5 = 40𝑡 + 20𝑡⃑𝑖 + 240𝑡 + 120𝑡⃑𝑗 + 10𝑡⃑𝑖 + 60𝑡⃑𝑗 = 40𝑡 + 30𝑡⃑𝑖 + 240𝑡 + 180𝑡 ⃑𝑗.

Поскольку единицы измерения, которые мы использовали до сих пор, — граммы и сантиметры в секунду, единицей измерения нашей силы будут дины. Мы можем сказать, что ⃑𝐹=40𝑡+30𝑡⃑𝑖+240𝑡+180𝑡⃑𝑗.дин

Теперь нам нужно найти горизонтальную составляющую этой силы, и нас интересует только ⃑𝑖 составляющая.Поэтому наше решение 40𝑡+30𝑡.

Разумно задаться вопросом, какой физический процесс мог произвести увеличение массы тела. Предположим, что тело аккумулирует некоторое количество вещества, которое он вступает в контакт с той частью среды, в которой движется тело. Масса накопленного вещества добавляется к массе тела.

Реальное представление такой ситуации потребует определения скорость, с которой изменялась площадь соприкосновения тела и среды а также скорость изменения скорости тела.Рассмотрение только этих переменных потребовало бы допущения, что ни плотность среды, ни процесс накопления вещества из среда менялась в зависимости от скорости тела или площади контакта тело и среда.

Рассмотрим пример, в котором различные процессы, воздействующие на накопление вещества телом изображаются упрощенно.

Пример 5. Нахождение силы, действующей на тело переменной массы в точке a Заданное время с использованием второго закона Ньютона

Шар массой 5 ​​г двигался прямолинейно через среду, наполненную пылью.Пыль скапливалась на его поверхности со скоростью 1 г/с. Найти модуль силы, действующей на мяч в момент времени 𝑡=5 секунд, если перемещение мяча выражается соотношением 𝑠(𝑡)=23𝑡+𝑡+7𝑡+1⃑𝑐,, где ⃑𝑐 — единичный вектор в направлении движения, а перемещение измеряется в сантиметрах.

Ответ

Сила, действующая на мяч, определяется выражением 𝐹=𝑚𝑣𝑡+𝑣𝑚𝑡.dddd

Мяч имеет массу 5 г при 𝑡=0, которая увеличивается со скоростью 1 г/с.Функция, представляющая изменение массы мяча со временем, 𝑚(𝑡), определяется выражением 𝑚(𝑡)=(1𝑡+5)=(𝑡+5).

Дифференцирование 𝑚(𝑡) дает нам дд𝑡(𝑡+5)=1.

Функция, представляющая скорость тела, определяется выражением 𝑣=𝑠𝑡=𝑡23𝑡+𝑡+7𝑡+1=2𝑡+2𝑡+7.dddd

Дифференцирование функции, представляющей 𝑣 по времени, дает нам dddd𝑣𝑡=𝑡2𝑡+2𝑡+7=(4𝑡+2).

Как произведение 𝑚 и 𝑣, 𝑚𝑣, следует дифференцировать по времени, мы можем использовать правило произведения, чтобы дифференцировать 𝑚𝑣 относительно 𝑡: 𝐹=𝑡(𝑚𝑣)=𝑚𝑣𝑡+𝑣𝑚𝑡.dddddd

Напоминая, что дд𝑚𝑡=1, у нас есть 𝐹=((𝑡+5)(4𝑡+2))+2𝑡+2𝑡+7.

Подстановка 𝑡=5 дает нам 𝐹=(10(22))+(50+10+7)𝐹=220+67=287⋅/.gcms

Масса находится в грамм а смещение в сантиметрах, и так сила исчисляется в ньютонах умножается на произведение количества сантиметры в метр и количество грамм в килограмме дать силу в ньютонах: 𝐹=2871010=287×10.N

Одна дина равна 10 с.ш., поэтому сила равна 287 дин.

Давайте теперь посмотрим на другой такой пример.

Пример 6. Нахождение скорости изменения массы шара при его движении в запыленной среде

Металлический шар движется прямолинейно с постоянной величиной скорости 1 м/с. Он попадает в запыленную среду. Если сила, действующая на шар в любой момент, равна 10 дин, найти скорость изменения массы шара за счет налипания пыли на его поверхность.

Ответ

Мяч имеет постоянную скорость.Итак, в формуле 𝐹=𝑚𝑣𝑡+𝑣𝑚𝑡,𝑣𝑡=0.dddddd

Тот факт, что скорость постоянна, позволяет нам выразить формулу как 𝐹=𝑣𝑚𝑡.dd

Скорость увеличения массы мяча прямо пропорциональна сила, действующая на мяч.

Сила, действующая на шар, равна 10 дин, где 1=1×1=10.dynegcmsN

Сила равна скорости изменения импульса мяча, выраженной в ньютонах по формуле 1010=10=𝑝𝑡=𝑡(𝑚𝑣).dddd

Скорость мяча 1 м/с, поэтому скорость изменения импульса можно записать как ддддд𝑝𝑡=1𝑚𝑡=𝑚𝑡.

Объединение значения силы с выражением для скорости изменения количества движения дает нам 10=𝑚𝑡,дд где 𝑚 находится в килограммы (кг), сила в ньютонах (Н), а скорость в метрах в секунду (РС). Скорость изменения массы, грамм в секунду (г/с), определяется как ddgs𝑚𝑡=1010=0,1/.

Ключевые моменты

  • Когда результирующая сила действует на тело постоянной массы, тело ускоряется в направлении действия силы. Величина ускорения зависит от величины силы и от массы тела по формуле 𝐹=𝑚𝑎, где 𝑚 – масса тела, а 𝑎 есть ускорение тела.
  • Импульс тела определяется выражением 𝑝=𝑚𝑣, где 𝑚 — масса тела, а 𝑣 — скорость тела.
  • Выражение второго закона Ньютона через скорость изменения импульса дает 𝐹=𝑝𝑡=𝑡(𝑚𝑣).dddd
    Используя правило произведения, это можно выразить как 𝐹=𝑚𝑣𝑡+𝑣𝑚𝑡.dddd
.

Оставить комментарий